SlideShare a Scribd company logo
1 of 13
Download to read offline
Management of Acute Kidney Injury: Core
Curriculum 2018
Peter K. Moore, Raymond K. Hsu, and Kathleen D. Liu
Acute kidney injury (AKI) is a heterogeneous disorder that is common in hospitalized patients and
associated with short- and long-term morbidity and mortality. When AKI is present, prompt workup
of the underlying cause should be pursued, with specific attention to reversible causes. Measures to
prevent AKI include optimization of volume status and avoidance of nephrotoxic medications.
Crystalloids are preferred over colloids for most patients, and hydroxyethyl starches should be
avoided. Volume overload in the setting of AKI is associated with adverse outcomes, so attention
should be paid to overall fluid balance. Currently there are no targeted pharmacotherapies approved
for the treatment of AKI. The optimal timing of renal replacement therapy in critically ill patients
with AKI is unclear, but is an area of active investigation. Recent studies suggest that AKI is not a
“self-limited” process, but is strongly linked to increased risk for chronic kidney disease, subsequent
AKI, and future mortality.
Introduction
Acute kidney injury (AKI) is a common
diagnosis in hospitalized patients, often
occurring in patients with multiple comorbid
conditions. It is associated with significant
increases in both short- and long-term
morbidity and mortality. Management of
AKI requires an in-depth understanding of
fluid and electrolyte homeostasis, as well as
appropriate use of renal replacement therapy
(RRT) in the acute setting. In this Core Cur-
riculum, we review key principles regarding
the diagnosis and general management of
AKI for clinicians. Specific management
of glomerulonephritis, thrombotic micro-
angiopathies, and AKI in the setting of
malignancy have been discussed in detail in
previous Core Curriculum articles and are not
addressed in depth.
Diagnosis and Evaluation of AKI
AKI is characterized by the accumulation of
creatinine, urea, and other unmeasured waste
products after an abrupt decrease in kidney
function. More recently, injury has replaced
“failure” to emphasize the disease continuum
because even modest reductions in kidney
function are associated with worse outcomes.
AKI is common, affecting approximately
5% to 10% of hospitalized patients and up to
60% of patients admitted to the intensive care
unit (ICU). AKI has a variety of causes and
should not be viewed as a single disease.
Although some management strategies can be
universally applied, others are disease specific.
Thus, when AKI is recognized, a critical
first step is the prompt evaluation for the cause
of AKI, with special attention to reversible
causes.
Definition
Due to significant heterogeneity in prior
studies, during the past 10 years efforts
have been made to develop consensus AKI
definitions, in particular for use in epidemi-
ologic studies and clinical trials. In 2012, the
KDIGO (Kidney Disease: Improving Global
Outcomes) group combined elements from
prior definitions (Table 1). Staging is based
on both urine output and serum creatinine
(Scr) concentration, and if urine output and
Scr concentration do not correspond to the
same stage, it has been recommended that
the highest stage should be considered.
However, although urine output is an
important kidney function parameter that
identifies patients at higher risk for adverse
outcomes, its pathophysiologic significance
in the absence of extremes of oliguria or
other surrogates of reduced glomerular
filtration rate (GFR) is more controversial. It
should be noted that patients who develop
AKI by KDIGO urine output criteria,
regardless of whether Scr criteria are present,
are at risk for developing fluid overload
given the typically high obligate intake of
critically ill patients.
FEATURE EDITOR:
Asghar Rastegar
ADVISORY BOARD:
Ursula C. Brewster
Michael Choi
Ann O’Hare
Manoocher Soleimani
The Core Curriculum
aims to give trainees
in nephrology a
strong knowledge
base in core topics in
the specialty by
providing an over-
view of the topic and
citing key references,
including the founda-
tional literature that
led to current clinical
approaches.
Complete author and article
information provided before
references.
Am J Kidney Dis. 72(1):
136-148. Published online
February 22, 2018.
doi: 10.1053/
j.ajkd.2017.11.021
© 2018 by the National
Kidney Foundation, Inc.
136 AJKD Vol 72 | Iss 1 | July 2018
Core Curriculum
Evaluation of Kidney Function in the Acute Care
Setting
At present, GFR is the gold-standard marker for acute
or chronic kidney disease, though it represents only one
of many affected functions. However, GFR is almost
never directly measured in the clinical setting, and sur-
rogate markers of kidney function are typically used.
Current eGFR equations (Cockcroft-Gault, MDRD Study,
and CKD-EPI) cannot be used when creatinine
concentration is not at steady state, as occurs during
AKI. Equations have been proposed to estimate kinetic
GFR when Scr concentration is actively changing, but
have not been validated for widespread use. In severe
AKI (eg, when the patient is oligoanuric), the
assumption should be that GFR is <10 mL/min when
urine output is minimal.
Furthermore, because Scr concentration lags acute changes
in kidney function, the current AKI stage may not reflect cur-
rent kidney function. Reductions in creatinine production
during acute illness and sarcopenia (which often develops with
prolonged illness), along with creatinine dilution during vol-
ume overload, further complicate the evaluation of kidney
function. Cystatin C has been used for GFR estimation and is
thought to be more accurate at higher GFRs and in those with
reduced muscle mass. However, the same limitations
regarding steady-state kinetics apply, and the impact of
volume of distribution has not been studied. There is consid-
erable interest in developing bedside tools for real-time
measuredGFR,butnosuchtoolsforclinicaluseexistatpresent.
Tubular injury biomarkers include neutrophil gelatinase-
associated lipocalin (NGAL), kidney injury molecule 1 (KIM-
1), interleukin 18 (IL-18), and liver-type fatty acid binding
protein (L-FABP). Future definitions of AKI may incorporate
biomarkers. There is also interest in biomarkers that reflect
kidney stress, including tissue injury metalloproteinase 2
(TIMP-2) and insulin-like growth factor binding protein 7
(IGFBP-7), which have recently been approved by the US
Food and DrugAdministration (FDA)fortheidentification of
patients at high risk for developing KDIGO stage 2 to 3 AKI
during the next 12 to 24 hours (these biomarkers are mar-
keted as the Nephrocheck Test [Astute Medical]). Our un-
derstanding of the clinical utility of this test is evolving
rapidly; at present, this test may be useful to identify patients
for implementation of care bundles (see below).
Additional Readings
► KIDGO AKI Workgroup. KDIGO clinical practice guideline for
AKI. Kidney Int Suppl. 2012;2:1-138. + ESSENTIAL READING
Table 1. Comparison of Recent Consensus AKI Definitions
AKI
Stage Urine Outputa
KDIGO AKIN RIFLE
1 <0.5 mL/kg/h for
6-12 h
Scr to 1.5-1.9 × baseline over
7 d or ≥0.3 mg/dL absolute
increase over 48 h
Scr to 1.5-2 × baseline
or ≥0.3 mg/dL absolute Scr
increase within 48 h
Risk: Scr to ≥1.5 × increase within
7 d, sustained for ≥24 h
2 <0.5 mL/kg/h for
≥12 h
Scr to 2.0-2.9 × baseline Scr to >2-3 × baseline Injury: Scr to ≥2 × increase
3 <0.3 mL/kg/h for ≥24
h or anuria for ≥12 h
Scr to ≥3.0 × baseline, or Scr
increase to ≥4.0 mg/dL or
initiation of RRT
Scr to >3.0 × baseline, or
Scr increase to ≥4.0 mg/dL
(with increase of 0.5 mg/dL)
or initiation of RRT
Failure: Scr to ≥3.0 × increase or
Scr increase to ≥4.0 mg/dL (with
increase of 0.5 mg/dL) or initiation
of RRT
Loss: Complete loss of kidney
function for >4 wk
ESKD: ESKD for >3 mo
Note: The first classification system, RIFLE, from the ADQI, incorporated 3 categories of injury and 2 outcomes that varied by severity. The outcomes (Loss, ESKD) were
eliminated from the subsequent AKIN and KDIGO definitions. The AKIN definition incorporated smaller changes in Scr concentration, and the KDIGO definition added
more definitive time frames to the definition. A key concept for the Scr-based definitions of AKI is the identification of baseline Scr concentration. Although the initial RIFLE
criteria recommended the use of an Scr concentration that would equate to eGFR of 75 mL/min/1.73 m2
by the MDRD Study equation (MDRD-75) if no baseline was
available, this definition does not account for chronic kidney disease if present. It is essential to look for a prior baseline/reference Scr concentration, ideally from the 365
days before hospital admission from a clinical context in which there was not concern for AKI (eg, a stable clinic visit). This concept is discussed in detail in the KDIGO AKI
clinical practice guideline.
Abbreviations: ADQI, Acute Dialysis Quality Initiative; AKI, acute kidney injury; AKIN, Acute Kidney Injury Network; eGFR, estimated glomerular filtration rate; ESKD,
end-stage kidney disease; KDIGO, Kidney Disease: Improving Global Outcomes; MDRD, Modification in Diet in Renal Disease; RIFLE, risk, injury, failure, loss of kidney
function, and end-stage kidney disease; RRT, renal replacement therapy; Scr, serum creatinine.
a
All 3 definitions (KDIGO, AKIN, RIFLE) use common urine output criteria.
Case: You are asked to see a 50-year-old African American
man with diabetes and known chronic kidney disease (CKD;
baseline Scr, 2.0 mg/dL) who is admitted with urosepsis.
Admission Scr concentration was 2.3 mg/dL and increased
to 2.6 mg/dL the following day.
Question 1: Which statement regarding his kidney
function is correct?
a) Using the MDRD (Modification of Diet in Renal Disease)
Study equation, his estimated GFR (eGFR) is 34 mL/min/
1.73 m2
b) Use of the CKD-EPI (CKD Epidemiology Collaboration)
equation is more appropriate for this patient, and his
eGFR is 32 mL/min/1.73 m2
c) Using the Cockcroft-Gault formula, his creatinine clear-
ance is 20 to 32 mL/min
d) His eGFR cannot be calculated because his Scr
concentration is not stable
For answer, see Appendix.
Core Curriculum
AJKD Vol 72 | Iss 1 | July 2018 137
► Kellum JA, Sileanu FE, Murugan R, Lucko N, Shaw AD, Clermont
G. Classifying AKI by urine output versus serum creatinine level. J
Am Soc Nephrol. 2015;26:2231-2238.
► Liu KD, Thompson BT, Ancukiewicz M, et al. AKI in patients with
acute lung injury: impact of fluid accumulation on classification of
AKI and associated outcomes. Crit Care Med. 2011;39:2665-
2671.
► Vijayan A, Faubel S, Askenazi DJ, et al. Clinical use of the urine
biomarker [TIMP-2] x [IGFBP7] for AKI risk assessment. Am J
Kidney Dis. 2016;68:19-28.
Cause and General Management of AKI
Evaluation of Cause of AKI
Those who meet criteria for AKI should have the cause
investigated, with special attention to treatable causes
(Table 2). Careful history taking, chart review, and phys-
ical examination remain the fundamental tenets of the
workup. For example, after cardiopulmonary bypass sur-
gery, AKI may be related to the bypass itself, hypovolemia,
postoperative cardiogenic or (rarely early on) septic shock,
or cholesterol emboli. Careful evaluation of the temporal
pattern of AKI relative to the surgery and other clinical
events (hypotension and iodinated contrast exposure), as
well as physical examination and laboratory findings (eg,
livedo reticularis and peripheral eosinophilia suggestive of
cholesterol emboli) are needed to differentiate these
conditions.
All patients with AKI need careful assessment of
hemodynamic and volume status using vital signs and
physical examination; critically ill patients, for example,
those in shock, may benefit from more invasive hemody-
namic monitoring (arterial line, central venous pressure, or
cardiac output monitoring). Urinary indexes (fractional
excretion of sodium and urea) may be helpful in
diagnosing decreased kidney perfusion (aka, prerenal
azotemia) if the patient is oligoanuric. However, the utility
of these indexes tends to be more limited in critically ill
adults, likely as a result of coexisting pre- and intrarenal
disease. Urinary microscopy for renal tubular epithelial cells
and granular casts may be helpful to make the concomitant
diagnosis of acute tubular necrosis (ATN), which is the
most common cause of AKI occurring in the hospital.
However, ATN is a misnomer because renal biopsy speci-
mens from patients with this clinical diagnosis tend to have
little frank necrosis and have evidence of significant
nonlethal cell injury. Thus, ATN is used clinically to
describe a specific and severe form of AKI that occurs from
a variety of causes (Box 1), rather than the pathology per se.
Table 2. Causes of AKI
Type Examples of Specific Causes
Decreased kidney perfusion (“prerenal” states)
Hypovolemia Increased losses (hemorrhage, burns, massive vomiting or diarrhea), poor oral intake
Reduced cardiac output Heart failure, cardiac tamponade, massive pulmonary embolism
Renal vasomodulation/shunting Medications (NSAID, ACEi/ARB, cyclosporine, iodinated contrast), hypercalcemia, hepatorenal
syndrome, abdominal compartment syndrome
Systemic vasodilation Sepsis, SIRS, hepatorenal syndrome
Intrarenal causes
Vascular Renal artery stenosis, arterial/venous cross-clamping
Microvascular Thrombotic microangiopathies (TTP, HUS, aHUS, DIC, APS, malignant hypertension,
scleroderma renal crisis, preeclampsia/HELLP syndrome, drug-induced), cholesterol emboli
Glomerular Rapidly progressive (crescentic) GN: anti–glomerular basement membrane; immune complex
diseases: IgA nephropathy, postinfectious, lupus, mixed cryoglobuminemia with MPGN; pauci-
immune glomerulonephritis: ANCA-associated vasculitides: GPA, MPA, EGPA (Churg-Strauss);
ANCA-negative; nephrotic-range proteinuria with associated AKI: HIV-associated nephropathy
(secondary FSGS); other causes of nephrotic-range proteinuria that commonly associate with
AKI: minimal change disease with ATN/AIN; membranous nephropathy + crescentic GN or renal
vein thrombosis; myeloma + multiple different pathologies, but in particular light chain cast
nephropathy
Tubulointerstitium AIN: medications, infection, lymphoproliferative disease; pigment nephropathy: rhabdomyolysis
(myoglobin), massive hemolysis (hemoglobin); crystal nephropathy: uric acid (tumor lysis),
acyclovir, sulfonamides, protease inhibitors (indinavir, azatanavir), methotrexate, ethylene glycol,
acute phosphate nephropathy, oxalate nephropathy; myeloma-associated AKI (cast
nephropathy); ATN: ischemia (shock, sepsis), inflammatory (sepsis, burns), medications
(see Box 1; osmotic nephrosis in setting of sucrose, mannitol and hydroxyethylstarch use)
Postrenal causes
Bladder outlet Benign prostatic hypertrophy, cancer, strictures, blood clots
Ureteral Bilateral obstruction (or unilateral with one kidney): stones, malignancy, retroperitoneal fibrosis
Renal pelvis Papillary necrosis (NSAIDs), stones
Note: Causes of AKI can be broadly divided into prerenal, intrarenal, and postrenal causes and then further subdivided as described.
Abbreviations: ACEi, angiotensin-converting enzyme inhibitor; (a)HUS, (atypical) hemolytic uremic syndrome; AKI, acute kidney injury; ANCA, antineutrophil cytoplasmic
antibody; APS, antiphospholipid syndrome; ATN/AIN, acute tubular necrosis/acute interstitial nephritis; ARB, angiotensin receptor blocker; DIC, disseminated intravas-
cular coagulation; EGPA, eosinophilic granulomatosis with polyangiitis; FSGS, focal segmental glomerulosclerosis; GN, glomerulonephritis; GPA, granulomatosis with
polyangiitis; HELLP, hemolysis, elevated liver enzymes, low platelet count) syndrome; HIV, human immunodeficiency virus; MPA, microscopic polyangiitis; MPGN,
membranoproliferative glomerulonephritis; NSAID, nonsteroidal anti-inflammatory drug; SIRS, systemic inflammatory response syndrome; TTP, thrombotic thrombocy-
topenic purpura.
138 AJKD Vol 72 | Iss 1 | July 2018
Core Curriculum
Patients suspected of having a specific treatable intra-
renal cause of AKI (such as acute interstitial nephritis
[AIN], glomerulonephritis, or thrombotic micro-
angiopathy) should have a urine sediment examination
and serologic/hematologic tests, as indicated. A kidney
biopsy should be considered when there is significant new
proteinuria (protein excretion > 3 g/d) or hematuria,
active urine sediment, or no readily identifiable cause of
decreased kidney perfusion, obstruction, or ATN. A
retrospective study of 68 critically ill patients who un-
derwent kidney biopsy based on clinical suspicion found
that 51% of patients had a specific cause of AKI, which led
to a significant change in treatment plan in 21%. However,
kidney biopsy was associated with complications in 22%,
most commonly from bleeding.
AIN is likely underdiagnosed in hospitalized patients
who develop AKI. The widespread use of antibiotics and
proton pump inhibitors puts these patients at higher
risk for AIN. When not accompanied by systemic symp-
toms (eg, rash and eosinophilia), AIN can be difficult to
diagnose. Urinary eosinophils have been demonstrated to
have poor test characteristics, and kidney biopsy is the only
definitive way to establish the diagnosis. Treatment of AIN
involves cessation of the culprit medication (if drug
induced) and consideration of steroid therapy.
Renal ultrasonography or computed tomography of the
abdomen and pelvis without iodinated contrast is indicated
when obstruction is suspected. In individuals with 2
kidneys, obstruction must be bilateral to cause AKI. In
those for whom there is another clear cause for AKI,
routine imaging may not be warranted.
Overview of AKI Management
Patients at risk for AKI and those with AKI should have
kidney function monitored closely by Scr concentration
and urine output (Fig 1). Careful assessment of volume
status and hemodynamics should be undertaken and
treated with intravenous fluids, diuretics, or other
means of hemodynamic support as indicated. These
treatments, along with RRT, are discussed in subsequent
sections.
Medications should be reviewed closely for nephrotoxic
agents, which should be discontinued or switched to med-
ications with less nephrotoxic potential. In a quality
improvement initiative, a pharmacy-led notification for
pediatric patients receiving 3 or more nephrotoxic medi-
cations or an aminoglycoside resulted in a 38% decrease in
nephrotoxic medication exposure and 64% decrease in AKI
incidence. In addition, medications that may accumulate
with reduced GFR should be avoided or adjusted, in
particular in patients with stage 2 or 3 AKI (Box 2). Although
not a medication per se, in patients with AKI, exposure to
gadolinium has been associated with nephrogenic systemic
fibrosis, a sclerosing condition of the skin and internal
organs that can result in death. Although the absolute inci-
dence of this condition is low, the relative risks and benefits
of gadolinium administration must be cautiously weighed.
Newer gadolinium preparations may be associated with a
lower risk of nephrogenic systemic fibrosis.
With regard to specific nephrotoxins, there is growing
interest in the nephrotoxic effects of vancomycin, which
in the setting of higher target trough concentrations for
severe methicillin-resistant Staphylococcus aureus (MRSA)
infections and declining kidney function can accumulate
to very high levels (>50 μg/mL). Casts that contain
nanospheric vancomycin have recently been described
in individuals with vancomycin-associated AKI. The
addition of piperacillin/tazobactam may potentiate the
nephrotoxicity of vancomycin, but the mechanism
is unclear. Nonsteroidal anti-inflammatory drugs and
angiotensin-converting enzyme inhibitors/angiotensin
receptor blockers are common classes of medications that
should be discontinued. Although recent studies suggest
Box 1. Medications Commonly Associated With Acute Tubular
Necrosis
• Aminoglycosides (tobramycin, gentamycin)
• NSAIDs (ibuprofen, naproxen, ketorolac, celecoxib)
• ACEi (captopril, lisinopril, benazepril, ramipril)
• ARB (losartan, valsartan, candesartan, irbesartan)
• Amphotericin
• Cisplatin
• Foscarnet
• Iodinated contrast
• Pentamidine
• Tenofovir
• Zolendronic acid
Note: Although not a classic cause of acute tubular necrosis, volume depletion
caused by diuretics can exacerbate the effects of some of these other medica-
tions. This table does not include common causes of pigment or crystal ne-
phropathy (which are described in Table 2) or medications associated with
osmotic injury.
Abbreviations: ACEi, angiotensin-converting enzyme inhibitor; ARB, angiotensin
receptor blocker; NSAIDs, nonsteroidal anti-inflammatory drugs.
Box 2. Key Medications Requiring Dose Adjustment (or
Cessation) in AKI
• Analgesics (morphine, meperidine, gabapentin, pregabalin)
• Antiepileptics (lamotrigine)
• Antivirals (acyclovir, gancyclovir, valgancyclovir)
• Antifungals (fluconazole)
• Antimicrobials (almost all antimicrobials need dose adjust-
ment in AKI, with important exceptions of azithromycin, cef-
triaxone, doxycycline, linezolid, moxifloxacin, nafcillin, rifampin)
• Diabetic agents (sulfonylureas, metformin)
• Allopurinol
• Baclofen
• Colchicine
• Digoxin
• Lithium
• Low-molecular-weight heparin
• NOACs
Note: Medications that are associated with acute tubular necrosis (Box 1) should
be withheld, if possible.
Abbreviations: AKI, acute kidney injury; NOAC, novel anticoagulants.
AJKD Vol 72 | Iss 1 | July 2018 139
Core Curriculum
that the association between iodinated radiocontrast and
AKI may not be as strong as previously thought, iodinated
contrast should be avoided if possible in patients with or
at risk for AKI.
A number of recent studies have examined bundled
protocols to improve the quality and consistency of care
for patients with or at risk for AKI. In one study, 276
patients undergoing cardiac surgery who had elevated
TIMP-2 × IGFBP-7 concentrations were randomly
assigned to routine care or a strictly implemented AKI
prevention protocol (from the KDIGO guideline and
consisting of items such as hemodynamic optimization
and avoidance of nephrotoxins). Postoperative AKI was
observed to be significantly lower in the protocol
group (55% vs 72%; absolute risk reduction,
17%; P = 0.004). Notably, the biomarker strategy
enriched for high-risk patients, reducing the number
needed to treat. However, more work is needed to
design and implement such potentially successful (and
sustainable) care bundles for AKI prevention and
management. Along the same lines, there has been
tremendous interest in the use of electronic alert sys-
tems to identify patients with early AKI or at high
risk for AKI, but the effectiveness of these alerts to
change clinical practice has been variable and limited to
date.
Additional Readings
► Augusto J-F, Lassalle V, Fillatre P, et al. Safety and diagnostic yield
of renal biopsy in the intensive care unit. Intensive Care Med.
2012;38:1826-1833.
► Darmon M, Ostermann M, Cerda J, et al. Diagnostic work-up and
specific causes of AKI. Intensive Care Med. 2017;43:829-840. +
ESSENTIAL READING
► Goldstein SL, Mottes T, Simpson K, et al. A sustained quality
improvement program reduces nephrotoxic medication-
associated AKI. Kidney Int. 2016;90:212-221.
► Kolhe NV, Reilly T, Leung J, et al. A simple care bundle for use in
AKI: a propensity score-matched cohort study. Nephrol Dial
Transplant. 2016;31:1846-1854.
► Lachance P, Villeneuve PM, Rewa OG, et al. Association between
E-alert implementation for detection of AKI and outcomes: a
systematic review. Nephrol Dial Transplant. 2017;32:265-272. +
ESSENTIAL READING
► Luque Y, Louis K, Jouanneau C, et al. Vancomycin-associated cast
nephropathy. J Am Soc Nephrol. 2017;28:1723-1728.
► Meersch M, Schmidt C, Hoffmeier A, et al. Prevention of cardiac
surgery-associated AKI by implementing the KDIGO guidelines in
high risk patients identified by biomarkers: the PREVAKI
randomized controlled trial. Intensive Care Med. 2017;43(11):
1551-1561.
► Moledina DG, Perazella MA. Drug-induced acute interstitial
nephritis. Clin J Am Soc Nephrol. 2017;12(12):2046-2049.
+ ESSENTIAL READING
► Perazella MA, Coca SG, Kanbay M, Brewster UC, Parikh CR.
Diagnostic value of urine microscopy for differential diagnosis of
AKI in hospitalized patients. Clin J Am Soc Nephrol.
2008;3:1615-1619.
Hemodynamic Support: Fluid Management and
Blood Pressure Targets
Management of hemodynamics in patients with AKI,
especially those in shock, is of critical importance.
Although under normal conditions relatively constant renal
blood flow can be maintained despite changes in blood
pressures through autoregulation, these mechanisms are
disrupted in AKI. Titration of fluids and vasopressors can
be complex: hypotension can result in continued kidney
damage in those with AKI, whereas administration of va-
sopressors in those without adequate intravascular volume
can further reduce renal blood flow. Conversely, patients
with AKI are at risk for volume overload, and intravenous
fluid loading may cause harm.
Intravenous Fluid Resuscitation
Outside the setting of iodinated contrast administration,
there are no randomized trials comparing intravenous
fluids to placebo for AKI prevention. However, it can be
assumed that those with reduced renal blood flow who can
augment their cardiac output by expansion of their intra-
vascular volume would benefit from fluid resuscitation.
Early goal-directed therapy, in which septic patients
received intravenous crystalloids, inotropes, and trans-
fusions according to predefined protocols, had no effect on
mortality or need for RRT in 3 subsequent large trials.
Although administration of intravenous fluids in patients
with sepsis and/or hypovolemia is beneficial initially, fluid
overload, especially in later disease, may confer harm.
Several retrospective studies have found associations
between positive fluid balance and mortality in critically ill
patients. In a large multicenter cohort focused on critically
ill patients, those with fluid overload (10% weight gain) at
the time of dialysis therapy initiation had an odds ratio
(OR) for death of 2.07 (95% confidence interval [CI],
1.27-3.37); findings were similar in those with AKI who
Case, continued: Review of the patient’s chart shows that
he has received 4 L of 0.9% saline solution intravenously in
the past 24 hours, and urine output has increased from 10 to
20 mL/h. On physical examination, vital signs include blood
pressure of 95/65 mm Hg, heart rate of 72 beats/min, and
oxygen saturation of 96% on 2 L/min by nasal cannula. His
lungs are clear. He has peripheral edema (2+).
Question 2: What would you recommend?
a) Continue with volume expansion because his urine output
has increased significantly
b) Add norepinephrine to increase his systolic blood pres-
sure to >105 mm Hg
c) Continue with volume expansion and add norepinephrine
as well
d) Start a trial of intravenous furosemide, which could help
manage his fluid overload
For answer, see Appendix.
140 AJKD Vol 72 | Iss 1 | July 2018
Core Curriculum
did not require dialysis. However, such analyses of fluid
overload are likely partially confounded by severity of
illness.
At present, there are numerous methods that can be
used to assess fluid responsiveness, and no one method can
be recommended above others. We recommend using
multiple clinical assessments and repeated measures to
assess fluid responsiveness. Intravenous fluids should be
used judiciously in patients with AKI who are not “volume
responsive.” After significant volume resuscitation, even if
patients remain volume responsive, vasopressor support
should be considered to avoid markedly positive fluid
balance. In those requiring volume resuscitation, the
choice of solution is controversial. Major trials of various
colloids, physiologic-balanced salt solutions, and saline
solution have been completed. We next review the evi-
dence base for fluid selection.
Colloid Versus Crystalloid
Colloids, such as albumin, hydroxyethyl starches (HESs),
and gelatins, rely on oncotic gradients to selectively
expand the intravascular space, while crystalloids equili-
brate across intravascular and extravascular spaces. Patients
with inflammatory states will have increased vascular
permeability, and some of this benefit may be lost.
Albumin appears to be a relatively safe, albeit more
expensive, alternative for resuscitation of critically ill pa-
tients. In the Saline Versus Albumin Fluid Evaluation
(SAFE) trial, ICU patients who received 4% albumin had
no renal or mortality benefit. However, less total volume
was required for resuscitation in the albumin group (2.2 vs
3.1 L). Given the reduction in volume needed, albumin
may have a role in special situations in which large vol-
umes of intravenous fluids are anticipated, such as septic
shock in a cirrhotic patient. There is a clear indication for
albumin in the setting of large-volume paracentesis for
patients with end-stage liver disease because albumin
infusion is associated with lower risk for AKI. Albumin
(and likely other colloids) should be avoided in patients
with traumatic brain injury due to an increased risk for
death.
There are a variety of HES preparations with differing
molecular weights, molar substitutions, and tonicities, all
of which are relatively inexpensive compared to albumin.
Several trials have demonstrated renal toxicity with
hyperoncotic HES administration due to proximal tubule
vacuolization and swelling (osmotic nephrosis). Subse-
quently, trials of iso-oncotic HES preparations have tested
the hypothesis that these preparations are less nephrotoxic.
The Crystalloid Versus Hydroxyethyl Starch (CHEST) Study
randomly assigned 7,000 ICU patients to receive saline
solution or an iso-oncotic 6% HES and found an increased
risk for RRT in the group that received HES (7.0% vs 5.3%;
P = 0.04). This study demonstrates one of the potential
challenges of the combined Scr concentration and urine
output–based AKI criteria: although there was more AKI
defined by RIFLE (risk, injury, failure, loss of kidney
function, and end-stage kidney disease) risk or injury in
the saline-solution arm, this was largely driven by urine
output. In contrast, the HES group, which had a lower
overall rate of AKI, had higher rates of RRT and a trend
toward more severe AKI. Following the publication of this
study, the FDA added additional warnings to the packaging
for HES.
The other synthetic colloids commonly used for volume
expansion are gelatins, but there are substantially fewer
data regarding the association of gelatins with AKI. In
general, given the lack of clear benefit with colloid
administration, routine use of these solutions is not
warranted.
Finally, there is still interest in the role of colloids for
the treatment of hypovolemic shock. Colloids Versus
Crystalloids for the Resuscitation of the Critically Ill
(CRISTAL) was a multicenter randomized open-label study
of more than 2,800 ICU patients with hypovolemic shock.
Patients were randomly assigned to fluid type (crystalloid
or colloid), and the selection of fluid was up to the study
investigator. About 45% of those in the colloid arm
received HES. There was no difference in RRT requirement
or mortality at 28 days (primary study end point). How-
ever, there was a significant reduction in mortality at 90
days, need for mechanical ventilation, and need for vaso-
pressors in those who received colloids. Thus, it has been
suggested that a key to the use of colloids is the optimal
timing of administration. Regardless, at present, there are
no data to support the routine use of colloid for volume
resuscitation.
Physiologic Balanced Salt Solution Versus Normal
Saline Solution
Isotonic 0.9% saline solution has a significantly higher
chloride content than the extracellular space in humans
(154 vs w110 mmol/L), and patients receiving normal
saline solution are at risk for hyperchloremic metabolic
acidosis. Hyperchloremia has been associated with
increased renal vascular resistance, increased renin activity,
and decreased GFR in animal studies. In healthy volunteers,
administration of 0.9% saline solution is associated with
increased extravascular volume and decreased renal cortical
tissue perfusion compared to a balanced salt solution.
To explore the hypothesis that chloride-rich fluids
increase the risk for AKI, Yunos et al performed an open-
labeled sequential study of ICU patients at a single
institution. During a 6-month period, patients were
administered balanced salt solutions for resuscitation and
were compared with controls from the corresponding 6
months from 1 year prior. The chloride-restricted group
had a lower incidence of AKI (8.4% vs 14%) and lower
rates of RRT (6.3% vs 10%). Subsequently, the SPLIT
(0.9% Saline vs Plasma-Lyte 148 for ICU Fluid Therapy)
trial, a multicenter randomized double-blind crossover
study, did not find a significant difference in rates of AKI,
AJKD Vol 72 | Iss 1 | July 2018 141
Core Curriculum
need for RRT, or mortality between the 0.9% saline so-
lution versus Plasma-Lyte groups (Baxter). However, this
study has been criticized because it was a predominantly
postoperative population that received only modest
resuscitation volumes (median, 2 L). Recently, data from
large pragmatic trials focused on patients admitted to the
emergency department or ICU at a single US institution
suggests benefit with balanced salt administration with
regard to the composite end point of Major Adverse Kid-
ney Events to day 30, defined as death, need for RRT or
persistently decreased kidney function at day 30/hospital
discharge (ClinicalTrials.gov identifiers NCT02444988,
NCT02547779, and NCT02614040).
Blood Pressure Management
There has been interest in optimal blood pressure tar-
gets in patients with shock. The SEPSISPAM (Sepsis and
Mean Arterial Pressure) trial randomly assigned patients
with septic shock requiring vasopressors to 2 blood
pressure goals, a standard mean arterial pressure (MAP)
goal (65-70 mm Hg) and a higher goal (80-85 mm
Hg). There was no difference in mortality between the
2 treatment groups. However, patients with chronic
hypertension in the higher MAP group had significantly
lower rates of AKI and RRT. The number needed to treat
to prevent 1 patient with hypertension from needing
RRT was modest, at 9.5. Patients in the higher MAP
group had higher rates of atrial fibrillation. Thus, blood
pressure targets should likely take into account pre-
morbid blood pressures, weighing the potential benefits
of increased renal perfusion against the potentially
deleterious effects of vasoconstriction resulting in
hypoperfusion of other organs.
Additional Readings
► Annane D, Siami S, Jaber S, et al. Effects of fluid resuscitation
with colloids vs crystalloids on mortality in critically ill patients
presenting with hypovolemic shock: the CRISTAL randomized
trial. JAMA. 2013;310:1809-1817.
► Asfar P, Meziani F, Hamel J-F, et al. High versus low blood-
pressure target in patients with septic shock. N Engl J Med.
2014;370:1583-1593.
► Bouchard J, Soroko SB, Chertow GM, et al. Fluid accumulation,
survival and recovery of kidney function in critically ill patients with
AKI. Kidney Int. 2009;76:422-427.
► Chowdhury AH, Cox EF, Francis ST, Lobo DN. A randomized,
controlled, double-blind crossover study on the effects of 2L in-
fusions of 0.9% saline and Plasma-Lyte(R) 148 on renal blood
flow velocity and renal cortical tissue perfusion in healthy
volunteers. Ann Surg. 2012;256:18-24.
► Davison D, Junker C. Advances in critical care for the nephrolo-
gist: hemodynamic monitoring and volume management. Clin J
Am Soc Nephrol. 2008;3:554-561. + ESSENTIAL READING
► Finfer S1, Bellomo R, Boyce N, French J, Myburgh J, Norton R;
SAFE Study Investigators. A comparison of albumin and saline for
fluid resuscitation in the ICU. N Engl J Med. 2004;350:2247-2256.
► Myburgh JA, Finfer S, Bellomo R, et al. Hydroxyethyl starch or
saline for fluid resuscitation in intensive care. N Engl J Med.
2012;367:1901-1911.
► Perner A, Prowle J, Joannidis M, Young P, Hjortrup PB, Pettil€a V.
Fluid management in AKI. Intensive Care Med. 2017;43:
807-815.
► PRISM Investigators; Rowan KM, Angus DC, Bailey M, et al.
Early, goal-directed therapy for septic shock — a patient-level
meta-analysis. N Engl J Med. 2017;376:2223-2234.
► Semler MW, Self W, Rice TW. Balanced crystalloids vs saline for
critically ill adults. Chest. 2017;152(4 suppl):A1120.
► Young P, Bailey M, Beasley R, et al. Effect of a buffered
crystalloid solution vs saline on AKI among patients in the ICU:
the SPLIT randomized clinical trial. JAMA. 2015;314:
1701-1710.
► Yunos NaM, Bellomo R, Hegarty C, Story D, Ho L, Bailey M.
Association between a chloride-liberal vs chloride-restrictive
intravenous fluid administration strategy and kidney injury in criti-
cally ill adults. JAMA. 2012;308:1566-1572.
Additional Therapies for AKI: Diuretics, Nutrition,
and the Future
Diuretics
Loop diuretics are commonly used in oliguric AKI despite
the lack of evidence for their benefit. In addition to pre-
venting volume overload, loop diuretics theoretically
attenuate ischemic tubular injury by decreasing metabolic
demand in the oxygen-poor renal medulla by inhibition of
the sodium/potassium/chloride (Na+
/K+
/2Cl-
) cotrans-
porter. However, clinical trials have failed to consistently
show a benefit of diuretics in AKI. Thus, KDIGO recom-
mends against the use of diuretics to treat AKI except in the
setting of volume overload, when they can be used for
management of volume overload itself. It has been pro-
posed that in early AKI, urine output response to loop
diuretics may have prognostic value. The hypothesis is that
patients with AKI who are able to augment urine output in
response to a diuretic challenge have intact tubular func-
tion and therefore may have a better renal prognosis.
However, this finding has not been validated in large
multicenter studies.
Nutrition and Glucose Control
AKI is a catabolic state, and patients with AKI may need
enteral or parenteral nutritional support. In general, the
enteral route is preferred due to the lower risk for infection
(and lower volumes needed to administer equivalent
calories). The nutrition prescription in AKI will vary
significantly depending on the underlying cause of AKI
and the form of RRT provided, if any. With regard to
glycemic control, the KDIGO guideline recommends
maintaining blood glucose concentrations between 110
and 149 mg/dL in critically ill patients, a range that has
never been formally evaluated in randomized trials. The
potential renal benefit of glucose control was demonstrated
in a single-center study of surgical patients randomly
assigned to a target blood glucose concentration of 80 to
110 mg/dL or 180 to 200 mg/dL, in which the incidence
of severe AKI and RRT was lower in the intensive arm
(4.8% vs 8.2%). A notable practice difference in this study
was the provision of dextrose in the immediate
142 AJKD Vol 72 | Iss 1 | July 2018
Core Curriculum
postoperative period, which may have increased the
adverse consequences of hyperglycemia. Subsequently, the
NICE-SUGAR (Normoglycemia in Intensive Care
Evaluation–Survival Using Glucose Algorithm Regulation)
Study, the largest randomized clinical trial of glycemic
control in critically ill patients, highlighted the potential
risks of intensive glycemic control. Participants (6,100 in
total) were randomly assigned to intensive (81-108 mg/
dL) or conventional (<180 mg/dL) glycemic control.
There was no difference in rates of RRT between groups.
However, intensive glycemic control was associated with
higher mortality (OR, 1.14; 95% CI, 1.02-1.28) and a
greater incidence of severe hypoglycemia (6.8% vs 0.5%).
Several additional clinical trials have had similar findings.
Patients with AKI may be at particularly high risk for severe
hypoglycemia given the kidney’s role in insulin meta-
bolism and glucose excretion. However, severe hypergly-
cemia is associated with increased morbidity and mortality
in a variety of clinical scenarios and should also be
avoided.
Pharmacotherapies for AKI
At this time, there are no pharmacologic therapies for
the prevention or treatment of AKI (Box 3). Because AKI
is a heterogeneous disease, identification of a single
therapy that will benefit all is challenging. Additionally,
the AKI insult almost always precedes AKI detection
and it is therefore difficult to intervene before the
disease is established. Early identification and treatment
of AKI with drugs that have pleiotropic effects on
multiple pathologic pathways are most likely to be
successful.
Additional Readings
► Chawla LS, Davison DL, Brasha-Mitchell E, et al. Development
and standardization of a furosemide stress test to predict the
severity of AKI. Crit Care. 2013;17:R207.
► NICE-SUGAR Study Investigators; Finfer S, Chittock DR, Su SY,
et al. Intensive versus conventional glucose control in critically ill
patients. N Engl J Med. 2009;360:1283-1297.
► Siew ED, Liu KD. Nutritional support in AKI. In: Mitch W, Ikizler TA,
eds. Handbook of Nutrition and the Kidney. Philadelphia, PA:
Wolters Kluwer; 2018.
► van den Berghe G, Wouters P, Weekers F, et al. Intensive insulin
therapy in critically ill patients. N Engl J Med. 2001;345:1359-
1367.
► Vanmassenhove J, Kielstein J, J€orres A, Biesen WV. Management
of patients at risk of AKI. Lancet. 2017;389:2139-2151.
+ ESSENTIAL READING
Management of Severe AKI, Including RRT
Patients with AKI may develop hyperkalemia, metabolic
acidosis, volume overload, and/or symptoms of uremia
due to reduced GFR. Hyperkalemia can be medically
managed as described in Table 3. Metabolic acidosis may
occur due to AKI itself (eg, inability to excrete organic
acids) or conditions associated with AKI (eg, hypo-
perfusion leading to lactic acidosis). The kidney plays an
important role, along with the liver, in lactate metabolism.
Treatment of metabolic acidosis depends on its severity
and must take into consideration absolute pH, rate of
change of acidosis, and its underlying cause. Metabolic
acidosis itself can be treated with bicarbonate or other base
equivalents. Diuretics can be used to manage volume
overload.
Despite these temporizing measures, some with severe
AKI will require RRT. The optimal timing of RRT is an area
of active investigation. Factors that may affect the timing of
RRT initiation are reviewed in the Continuous Dialysis
Therapies Core Curriculum. With regard to available
data, retrospective studies have showed an association
between early RRT and favorable outcomes. However,
Box 3. Agents Tested in Selected Trials for Treatment of AKI
Trials ongoing
• Alkaline phosphatase (sepsis-associated AKI)
• L-Carnitine (sepsis-associated AKI)
• Remote ischemic preconditioning (post operative AKI)
• p53-targeted siRNA (post–cardiac surgery AKI)
• Extracorporeal devices (dialysis-requiring AKI)
• Vitamin D (hospitalized AKI)
• Uremic toxin absorption/pentoxifylline (hospital-acquired AKI)
No clear evidence of benefit
• α-Melanocyte-stimulating hormone
• Atrial natriuretic peptide
• Calcium channel blockers
• Diureticsa
• Dopamine
• Erythropoietin
• Fenoldopam
• Insulin growth factor
• N-Acetylcysteine
• Statins
• Aminophylline/theophyllineb
Abbreviations: AKI, acute kidney injury; siRNA, short interfering RNA.
a
Potentially useful for volume management, but not for treatment of AKI.
b
Some interest remains for AKI prevention in neonates.
Case, continued: Two days later, the patient remains oli-
guric despite a trial of furosemide, and his Scr concentration
has increased to 5.5 mg/dL. Blood pressure is 105/75 mm
Hg. He has peripheral edema (3+), and oxygen saturation is
91% on 5 L/min by nasal cannula. You plan to initiate RRT.
Question 3: Which of the following is the best state-
ment with respect to this set of circumstances?
a) Given the role of sepsis in development of his AKI,
continuous RRT (CRRT) is preferable to intermittent he-
modialysis (IHD) for this patient
b) If CRRT is selected, the prescribed dose should be 35 to
40 mL/kg/h
c) CRRT and IHD have similar clinical outcomes
d) RRT can be postponed until the patient develops clear
signs of uremia
For answer, see Appendix.
AJKD Vol 72 | Iss 1 | July 2018 143
Core Curriculum
many of these studies were limited in their assessment of
“early” based on serum urea nitrogen or creatinine con-
centration without other clinical information. Recently, 2
trials have assessed the impact of RRT timing in ICU pa-
tients. The ELAIN (Early Versus Late Initiation of Renal
Replacement Therapy in Critically Ill Patients With Acute
Kidney Injury) trial found that 90-day mortality was
significantly lower in patients randomly assigned to earlier
RRT. However, this study has been criticized as a single-
center trial that included many post–cardiac surgery pa-
tients and enrolled patients with early AKI (KDIGO stage 2
AKI and elevated plasma NGAL). In contrast, the AKIKI
(Artificial Kidney Initiation in Kidney Injury) Study was a
multicenter trial that randomly assigned patients with
more severe (KDIGO stage 3) AKI and did not find a dif-
ference in mortality between early and delayed RRT. Thus,
the questions of whether early RRT is beneficial, and if so,
in which patients, remain unanswered. Two large ongoing
trials will help answer these questions (ClinicalTrials.gov
identifiers NCT01682590 and NCT02568722). Interest-
ingly, in the STARRT-AKI (Standard vs. Accelerated Initi-
ation of RRT in Acute Kidney Injury) pilot and in AKIKI, a
significant proportion of participants in the late initiation
arm recovered kidney function before RRT. This finding
suggests that in addition to patients who may benefit from
the earlier provision of RRT, there are patients who may
recover before the need for RRT, and our ability to identify
these patients is limited at best.
RRT Prescription, Including Modality and Dose
Several aspects of the RRT prescription, including site
selection for vascular access, choice of membrane and anti-
coagulation, and differences between convective
and diffusive clearance, are discussed in detail in the
Continuous Dialysis Therapies Core Curriculum. With regard
to modality, the most widely used are CRRT and IHD. Pro-
longed intermittent RRT/sustained low-efficiency dialysis
are additional options that are used less frequently. Although
they appear to have similar outcomes in preliminary studies
and meta-analyses comparing these modalities with other
forms of RRT for AKI, there is a need for better quality evi-
dence in these areas before their routine use can be recom-
mended. Peritoneal dialysis can be used in the acute setting as
well and can be of particular use in resource-limited settings.
There has been much interest in whether CRRT is
associated with more favorable outcomes, including lower
mortality and enhanced renal recovery. However, to date,
small randomized clinical trials and meta-analyses have
found no association between modality and outcome
(mortality or renal recovery). Thus, as recommended by
the KDIGO guideline, CRRT and IHD are complementary
therapies; treatment considerations include the individual
patient’s hemodynamic status, degree of volume overload
and bleeding risk, and the treating facility’s availability/
experience.
An early single-center trial suggested that patients with
higher CRRT intensity (35 or 45 mL/kg/h) had lower
mortality when compared to lower intensity (20 mL/kg/
h). However, 2 subsequent multicenter randomized
controlled trials, the VA/NIH ATN (Veterans Affairs/Na-
tional Institutes of Health Acute Renal Failure Trial
Network) and ANZICS RENAL (Australian and New Zea-
land Intensive Care Society Randomised Evaluation of
Normal Versus Augmented Level of Renal Replacement
Therapy in ICU) studies, found that there was no differ-
ence in mortality or renal recovery when comparing high-
to low-intensity RRT. Patients in the high-intensity
arm were more likely to have hypophosphatemia.
Table 3. Medical Management of Hyperkalemia
Purpose of Treatment Drug Usual Dose Notes
Stabilization of cardiac
membrane
Calcium gluconate
or calcium chloride
1 g IV over minutes,
repeat as needed
Given when ECG changesa
present; use with
caution with digoxinb
Transcellular potassium
ion shift
Insulin (regular) 10 U IV or weight basedc
Typically administered with 25-50 g of IV glucose
β2-adrenergic agonist Albuterol 5-20 mg,
nebulized
Watch for tachycardia
Sodium bicarbonate
(NaHCO3)
50 mEq/50 mL IV Controversial outside of setting of severe
metabolic acidosis; bolus dose is very hypertonic
with significant sodium load
Removal from body Loop diuretic Furosemide 40-60 mg IV Supplement with isotonic saline solution if
euvolemic or hypovolemic; highly effective if the
patient is diuretic-responsive
Cation exchange
resins
Sodium polystyrene
sulfonate, 15 g orally/
rectally, 1-4×/d
Use with caution with ileus/obstruction or in
patients with dehydration; associated with risk for
colonic necrosis; need to ensure that the resin
transits out of the GI tract; other resins are under
development but are not approved for use for
acute hyperkalemia
Abbreviations: AKI, acute kidney injury; ECG, electrocardiographic; GI, gastrointestinal; IV, intravenous.
a
ECG changes include peaked T waves, prolongation of PR interval, widening of QRS, second- or third-degree heart block, and sine wave pattern (from least severe to
most life-threatening).
b
For AKI with hyperkalemia in the setting of digoxin toxicity, reversal of digoxin toxicity with digoxin antibody fragments is the treatment of choice.
c
Weight-based insulin dosing is 0.1 U/kg of body weight, up to 10 units. It is associated with reduced risk for hypoglycemia without affecting potassium-lowering effect.
144 AJKD Vol 72 | Iss 1 | July 2018
Core Curriculum
Consequently, current guidelines recommend goal effluent
flow rates of 20 to 25 mL/kg/h.
With regard to IHD dosing, it is important to routinely
check the urea reduction ratio or Kt/V to ensure that
dialysis is adequate. In the VA/NIH ATN study, median
duration of an IHD session was 4 hours, with a mean
blood flow rate of 360 mL/min, highlighting that in these
catabolic patients, substantial time is needed to ensure an
adequate dialysis dose.
Discontinuation of RRT
The decision to discontinue RRT in patients with AKI is
made based on 1 of 3 clinical scenarios: intrinsic kidney
function has adequately improved to meet demands, the
disorder that prompted renal support has improved, or
continued RRT is no longer consistent with goals of care.
There is no definitive prospective evidence to guide cli-
nicians, but urine output appears to be predictive of
successful RRT discontinuation. In one study of patients on
CRRT, 24-hour urine output > 400 mL/d in patients not
using diuretics or >2,300 mL/d in patients using diuretics
had >80% chance of successful RRT discontinuation. Other
studies have suggested that quantitation of timed urinary
creatinine and urea excretion (either as total excretion per
24-hour period or calculation of creatinine and urea
clearance) may be helpful. Prospective studies are needed
to help guide clinicians on when to attempt RRT
discontinuation.
Additional Readings
► Gaudry S, Hajage D, Schortgen F, et al. Initiation strategies for
renal-replacement therapy in the intensive care unit. N Engl J
Med. 2016;375:122-133.
► Macedo E, Mehta RL. Continuous dialysis therapies: core cur-
riculum 2016. Am J Kidney Dis. 2016;68:645-657. + ESSENTIAL
READING
Figure 1. Conceptual model for the diagnosis and treatment
of acute kidney injury (AKI). Abbreviations: UO, urine output;
RRT, renal replacement therapy; sCr, serum creatinine.
AJKD Vol 72 | Iss 1 | July 2018 145
Core Curriculum
► Ponce D, Balbi A, Cullis B. Acute PD: evidence, guidelines, and
controversies. Semin Nephrol. 2017;37:103-112.
► RENAL Replacement Therapy Study Investigators’ Bellomo R,
Cass A, Cole L, et al. Intensity of continuous renal-replacement
therapy in critically ill patients. N Engl J Med. 2009;361:
1627-1638.
► Uchino S, Bellomo R, Morimatsu H, et al. Discontinuation of
continuous renal replacement therapy: a post hoc analysis of a
prospective multicenter observational study. Crit Care Med.
2009;37:2576-2582.
► VA/NIH Acute Renal Failure Trial Network; Palevsky PM, Zhang
JH, O’Connor TZ, et al. Intensity of renal support in critically ill
patients with AKI. N Engl J Med. 2008;359:7-20. + ESSENTIAL
READING
► Wald R, Adhikari NK, Smith OM, et al. Comparison of standard
and accelerated initiation of renal replacement therapy in AKI.
Kidney Int. 2015;88:897-904.
► Wheeler DT, Schafers SJ, Horwedel TA, Deal EN, Tobin GS.
Weight-based insulin dosing for acute hyperkalemia results in less
hypoglycemia. J Hosp Med. 2016;11:355-357.
► Zarbock A, Kellum JA, Schmidt C, et al. Effect of early vs delayed
initiation of renal replacement therapy on mortality in critically ill
patients with AKI: the ELAIN randomized clinical trial. JAMA.
2016;315:2190-2199.
Contrast-Induced AKI
Contrast-induced AKI (CI-AKI, also referred to as contrast-
associated AKI) is a specific form of AKI that usually
manifests as a transient small increase in Scr concentration
within a few days of exposure to intravascular iodinated
contrast. Despite its usually self-limited course, CI-AKI is
associated with increased short- and long-term mortality,
as well as progressive CKD. Recently, the degree to which
radiocontrast affects the kidney has been debated because
several studies (both meta-analyses and cohort studies)
have suggested that in the aggregate population, the risk
for AKI after contrast administration is perhaps
overemphasized.
Nonetheless, in clinical practice, for any given study
requiring iodinated contrast, the potential risks and
benefits should be weighed closely. Along the same
lines, patient- and procedure-level factors contribute to
the risk for CI-AKI and should be assessed. The primary
risk factor for CI-AKI is CKD, and the incidence of CI-
AKI increases incrementally as GFR decreases or pro-
teinuria/albuminuria increases. Diabetes further in-
creases the risk in those with CKD. Additional patient-
specific risk factors include low effective circulating
blood volume and nonsteroidal anti-inflammatory drug
use. Procedure-related risk factors include higher
contrast volume, intra-arterial procedures, multiple
contrast exposures in a short interval, and hyperosmolar
contrast agents.
Management of CI-AKI aims primarily at prevention.
Consideration should be given to alternative noncontrast
studies if possible. Those who undergo iodinated
contrast studies should have treatment with nonste-
roidal anti-inflammatory drugs and other nephrotoxins
discontinued, ideally at least 24 hours before the pro-
cedure. Low- or iso-osmolar radiocontrast should be
used, at the lowest possible volume required. Isotonic
intravenous fluid administration reduces the risk for CI-
AKI and should be used in those at elevated risk. Typical
regimens consist of a 1-mL/kg/h infusion 12 hours
before and 12 hours after contrast exposure, or 3 mL/kg/
h 1 hour before and 1.5 mL/kg/h for 4 to 6 hours
postprocedure. With regard to fluid selection, although
small studies suggested a benefit to the use of isotonic
sodium bicarbonate solution, a large randomized clinical
trial of isotonic bicarbonate versus normal saline solution
(factorialized with N-acetylcysteine vs placebo) in high-
risk patients undergoing angiography showed no benefit
with bicarbonate or N-acetylcysteine with regard to a
composite end point of death, RRT, and 50% reduction in
GFR at 90 days. There have been a variety of other
pharmacotherapies evaluated for CI-AKI prevention, none
of which is clearly beneficial. Hemodialysis after admin-
istration of contrast is ineffective for preventing CI-AKI
and may cause harm.
Additional Readings
► James MT, Samuel SM, Manning MA, et al. Contrast-induced AKI
and risk of adverse clinical outcomes after coronary angiography:
a systematic review and meta-analysis. Circ Cardiovasc Interv.
2013;6:37-43.
► Weisbord SD, Gallagher M, Jneid H, et al. Outcomes after
angiography with sodium bicarbonate and acetylcysteine
[published online ahead of print November 12, 2017].
N Engl J Med. doi: 10.1056/NEJMoa1710933. + ESSENTIAL
READING
► Weisbord SD, Palevsky PM. Prevention strategies for contrast-
induced nephropathy. Ann Intern Med. 2016;164:511.
+ ESSENTIAL READING
► Wilhelm-Leen E, Montez-Rath ME, Chertow G. Estimating the risk
of radiocontrast-associated nephropathy. J Am Soc Nephrol.
2017;28:653-659.
Long-term Consequences of AKI
Case, continued: Two weeks later, the patient begins to
recover kidney function. He is discharged from the hospital
with an Scr concentration that is stable at 2.5 mg/dL. He
asks you about the long-term impact of the AKI on his health.
Question 4: What is the best way to respond to his
stated concern?
a) Because this was an acute event due to urosepsis, which
is now fully treated, the AKI has no meaningful impact on
the course of his underlying CKD
b) His risk for future dialysis dependency has increased
significantly after this episode of AKI
c) There is no association between his recent AKI and risk
for future cardiovascular disease
d) He should expect further recovery of his kidney function
and return to his baseline over the next few months
For answer, see Appendix.
146 AJKD Vol 72 | Iss 1 | July 2018
Core Curriculum
Although previously it was believed that most pa-
tients who developed AKI fully recovered, it is now
recognized that those who experience AKI have
increased risk for subsequent AKI, progressive CKD, and
future mortality. Even mild stages of AKI are associated
with incident CKD. In a propensity-matched cohort
study of hospitalized patients who experienced renal
recovery based on Scr concentration, those with AKI had
an increased rate of incident CKD (relative risk [RR],
2.14; 95% CI, 1.96-2.43) and mortality (RR, 1.48; 95%
CI, 1.20-1.83).
A pooled analysis of studies of long-term risk for
CKD and dialysis dependence found a pooled hazard
ratio of 8.8 for CKD and 3.1 for end-stage kidney dis-
ease in patients with AKI compared with those without
AKI. There was a graded increase in risk by severity of
AKI. Given the retrospective nature of these associa-
tions, it is controversial whether this is a causal rela-
tionship or the development of AKI is simply a marker
of those at higher risk for CKD. An ongoing matched
cohort study sponsored by the National Institute
of Diabetes and Digestive and Kidney Diseases (NIDDK)
is focusing on individuals who survive 3 months
after a hospitalization with or without AKI and is
designed to try to address some of these remaining
questions.
Furthermore, identifying renal recovery based on Scr
concentration may be difficult because hospitalized
patients are at risk for muscle mass loss, creatinine
production can by decreased by inflammation, and Scr
can by diluted by iatrogenic volume overload. This was
demonstrated by Prowle et al, who found that Scr
concentrations were lower on discharge than on
admission in ICU patients without AKI. Using a model
taking into account this decrease in Scr concentration,
significantly more patients with AKI would have had
continued decreased kidney function compared with
estimates calculated from unadjusted discharge Scr
concentrations.
Apart from CKD and death, there has been considerable
interest in AKI as a risk factor for cardiovascular disease
events. A recent meta-analysis showed that AKI was
associated with a 58% increased risk for subsequent heart
failure events and 40% increased risk for acute myocardial
infarction. However, because most studies were con-
ducted in patients with pre-existing cardiovascular disease,
further research is needed to elucidate potential mecha-
nisms by which AKI contributes to CVD. One potential
mechanism is through hypertension. A recent study of
more than 40,000 hospitalized adult patients without
known hypertension showed that an episode of in-
hospital AKI was strongly predictive of subsequent hy-
pertension within 2 years (adjusted OR, 1.22; 95% CI,
1.12-1.33).
It is currently recommended that all patients who
experience AKI have their kidney function re-evaluated 3
months after AKI to identify those with new/worsening
CKD, which should be managed accordingly. Even those
who return to their baseline kidney function should be
considered at elevated risk for the development of CKD. At
this time it is unclear whether any intervention or increase
in monitoring would reduce the risk for poor outcomes in
these patients.
Additional Readings
► Bucaloiu ID, Kirchner HL, Norfolk ER, Hartle JE, Perkins RM.
Increased risk of death and de novo CKD following reversible AKI.
Kidney Int. 2012;81:477-485. + ESSENTIAL READING
► Chawla LS, Eggers PW, Star RA, Kimmel PL. AKI and CKD as
interconnected syndromes. N Engl J Med. 2014;371:58-66.
+ ESSENTIAL READING
► Coca SG, Singanamala S, Parikh CR. CKD after AKI: a system-
atic review and meta-analysis. Kidney Int. 2012;81:442-448.
► Hsu CY, Hsu RK, Yang J, Ordonez JD, Zheng S, Go AS. Elevated
BP after AKI. J Am Soc Nephrol. 2016;27:914-923.
► Odutayo A, Wong CX, Farkouh M, et al. AKI and long-term risk for
cardiovascular events and mortality. J Am Soc Nephrol.
2017;28:377-387.
► Parr SK, Siew ED. Delayed consequences of AKI. Adv Chronic
Kidney Dis. 2016;23:186-194.
► Prowle JR, Kolic I, Purdell-Lewis J, Taylor R, Pearse RM, Kirwan
CJ. Serum creatinine changes associated with critical illness and
detection of persistent renal dysfunction after AKI. Clin J Am Soc
Nephrol. 2014;9:1015-1023.
Article Information
Authors’ Full Names and Academic Degrees: Peter K. Moore,
MD, Raymond K. Hsu, MD, MAS, and Kathleen D. Liu, MD, PhD,
MAS.
Authors’ Affiliations: Division of Hospital Medicine, Department of
Medicine, San Francisco Veterans Affairs Medical Center and
University of California San Francisco (PKM); and Division of
Nephrology, Department of Medicine (RKS, KDL), and Critical
Care Medicine, Department of Anesthesia (KDL), University of
California, San Francisco, CA.
Address for Correspondence: Kathleen D. Liu, MD, PhD, MAS,
Division of Nephrology, Box 0532, University of California, San
Francisco, San Francisco, CA 94143-0532.. E-mail: kathleen.liu@
ucsf.edu
Support: None.
Financial Disclosure: Dr Hsu reports funding from the
NIDDK and has been a consultant for Retrophin. Dr Liu
has funding from the NIDDK and the National Heart, Lung
and Blood Institute; has been a consultant for
Achaogen, Durect, Quark, Potrero Medical and Theravance;
has served on an Advisory Board for ZS Pharma; and holds
stock in Amgen. None of these financial disclosures are
relevant to the current article.
Acknowledgements: The authors appreciate the assistance of
Dr Asghar Rastegar with the case that is part of this Core
Curriculum.
Peer Review: Received August 3, 2017 in response to an
invitation from the journal. Evaluated by 3 external peer
reviewers, with direct editorial input from the Feature Editor, the
Education Editor, and a Deputy Editor. Accepted in revised form
November 19, 2017.
AJKD Vol 72 | Iss 1 | July 2018 147
Core Curriculum
APPENDIX
Answer to Question 1: (d) Because his Scr concentration
is increasing, none of the standard formulas should be
used to estimate his kidney function.
Answer to Question 2: He is fluid overloaded, as shown
by his markedly positive fluid balance and the presence of
edema. At this point, additional fluid should be adminis-
tered with caution because it will likely only exacerbate
fluid overload. MAP is 75 mm Hg and therefore the
addition of a vasopressor is not justified. Use of furosemide
may increase urine output and decrease fluid overload;
however, it probably would not change the overall clinical
outcome. Thus, (d) is the best answer.
Answer to Question 3: (c) There is no evidence that CRRT
has a special role in patients with sepsis-associated AKI.
With regard to CRRT dose, 20 to 25 mL/kg/h has similar
outcomes to higher doses of therapy. The major indication
for RRT is the lack of renal recovery in association with
significant fluid overload and progressive hypoxemia.
Answer to Question 4: (b) AKI is a risk factor for CKD
progression and end-stage kidney disease, as well as car-
diovascular events (see text for full discussion). It is un-
known whether he will have further improvement in
kidney function over time, but it seems unlikely because
his Scr concentration is now stable.
148 AJKD Vol 72 | Iss 1 | July 2018
Core Curriculum

More Related Content

What's hot

A New Perspective on AKI
A New Perspective on AKIA New Perspective on AKI
A New Perspective on AKISteve Chen
 
Pentoxyfilline in Diabetic Renal Disease and Renal Transplantation
Pentoxyfilline in Diabetic Renal Disease and Renal TransplantationPentoxyfilline in Diabetic Renal Disease and Renal Transplantation
Pentoxyfilline in Diabetic Renal Disease and Renal TransplantationChristos Argyropoulos
 
Acute kidney injury in critically ill patients in the new millenium: definiti...
Acute kidney injury in critically ill patients in the new millenium: definiti...Acute kidney injury in critically ill patients in the new millenium: definiti...
Acute kidney injury in critically ill patients in the new millenium: definiti...Vall d'Hebron Institute of Research (VHIR)
 
Relative Blood Volume Monitoring and Applications in Dialysis
Relative Blood Volume Monitoring and Applications in DialysisRelative Blood Volume Monitoring and Applications in Dialysis
Relative Blood Volume Monitoring and Applications in DialysisChristos Argyropoulos
 
Steroid Withdrawal after kidney transplantation
Steroid Withdrawal after kidney transplantationSteroid Withdrawal after kidney transplantation
Steroid Withdrawal after kidney transplantationChristos Argyropoulos
 
Gpc manejo de enfermedades glomerulares-kdigo-2021
Gpc manejo de enfermedades glomerulares-kdigo-2021Gpc manejo de enfermedades glomerulares-kdigo-2021
Gpc manejo de enfermedades glomerulares-kdigo-2021WilliamBarrera34
 
Anemia management in CKD (ESA Therapy) - Dr. Gawad
Anemia management in CKD (ESA Therapy) - Dr. GawadAnemia management in CKD (ESA Therapy) - Dr. Gawad
Anemia management in CKD (ESA Therapy) - Dr. GawadNephroTube - Dr.Gawad
 
Marginal Donors/Expanded Criteria Donors Kidneys
Marginal Donors/Expanded Criteria Donors KidneysMarginal Donors/Expanded Criteria Donors Kidneys
Marginal Donors/Expanded Criteria Donors KidneysAbdullah Ansari
 
CKD MBD - Drug Related Issues - Dr. Gawad
CKD MBD - Drug Related Issues - Dr. GawadCKD MBD - Drug Related Issues - Dr. Gawad
CKD MBD - Drug Related Issues - Dr. GawadNephroTube - Dr.Gawad
 
NGAL in AKI
NGAL in AKINGAL in AKI
NGAL in AKIAla Ali
 
Steroid Sparing Regimens in Kidney Transplantation
Steroid Sparing Regimens in Kidney TransplantationSteroid Sparing Regimens in Kidney Transplantation
Steroid Sparing Regimens in Kidney TransplantationAbdullah Ansari
 
Aki biomarker 2016 chaken maniyan
Aki biomarker 2016 chaken maniyanAki biomarker 2016 chaken maniyan
Aki biomarker 2016 chaken maniyanCHAKEN MANIYAN
 
Salon b 13 kasim 15.45 17.00 müge aydoğdu-ing
Salon b 13 kasim 15.45 17.00 müge aydoğdu-ingSalon b 13 kasim 15.45 17.00 müge aydoğdu-ing
Salon b 13 kasim 15.45 17.00 müge aydoğdu-ingtyfngnc
 
When to initiate RRT in patients with AKI - Does timing matter?
When to initiate RRT in patients with AKI - Does timing matter?When to initiate RRT in patients with AKI - Does timing matter?
When to initiate RRT in patients with AKI - Does timing matter?Apollo Hospitals
 
2016: Geriatric Nephrology - Beben
2016: Geriatric Nephrology - Beben2016: Geriatric Nephrology - Beben
2016: Geriatric Nephrology - BebenSDGWEP
 

What's hot (20)

A New Perspective on AKI
A New Perspective on AKIA New Perspective on AKI
A New Perspective on AKI
 
Pentoxyfilline in Diabetic Renal Disease and Renal Transplantation
Pentoxyfilline in Diabetic Renal Disease and Renal TransplantationPentoxyfilline in Diabetic Renal Disease and Renal Transplantation
Pentoxyfilline in Diabetic Renal Disease and Renal Transplantation
 
Acute kidney injury in critically ill patients in the new millenium: definiti...
Acute kidney injury in critically ill patients in the new millenium: definiti...Acute kidney injury in critically ill patients in the new millenium: definiti...
Acute kidney injury in critically ill patients in the new millenium: definiti...
 
Relative Blood Volume Monitoring and Applications in Dialysis
Relative Blood Volume Monitoring and Applications in DialysisRelative Blood Volume Monitoring and Applications in Dialysis
Relative Blood Volume Monitoring and Applications in Dialysis
 
Steroid Withdrawal after kidney transplantation
Steroid Withdrawal after kidney transplantationSteroid Withdrawal after kidney transplantation
Steroid Withdrawal after kidney transplantation
 
Kdigo aki guideline
Kdigo aki guidelineKdigo aki guideline
Kdigo aki guideline
 
Gpc manejo de enfermedades glomerulares-kdigo-2021
Gpc manejo de enfermedades glomerulares-kdigo-2021Gpc manejo de enfermedades glomerulares-kdigo-2021
Gpc manejo de enfermedades glomerulares-kdigo-2021
 
Anemia management in CKD (ESA Therapy) - Dr. Gawad
Anemia management in CKD (ESA Therapy) - Dr. GawadAnemia management in CKD (ESA Therapy) - Dr. Gawad
Anemia management in CKD (ESA Therapy) - Dr. Gawad
 
Dr obrador nc ds ckd prev & keep mexico v2 ac 1 25-13
Dr obrador nc ds ckd prev & keep mexico v2 ac 1 25-13Dr obrador nc ds ckd prev & keep mexico v2 ac 1 25-13
Dr obrador nc ds ckd prev & keep mexico v2 ac 1 25-13
 
Marginal Donors/Expanded Criteria Donors Kidneys
Marginal Donors/Expanded Criteria Donors KidneysMarginal Donors/Expanded Criteria Donors Kidneys
Marginal Donors/Expanded Criteria Donors Kidneys
 
Bariatric Surgery and Kidney Stones
Bariatric Surgery and Kidney StonesBariatric Surgery and Kidney Stones
Bariatric Surgery and Kidney Stones
 
CKD MBD - Drug Related Issues - Dr. Gawad
CKD MBD - Drug Related Issues - Dr. GawadCKD MBD - Drug Related Issues - Dr. Gawad
CKD MBD - Drug Related Issues - Dr. Gawad
 
Dialysis in acute kidney injury
Dialysis in acute kidney injuryDialysis in acute kidney injury
Dialysis in acute kidney injury
 
NGAL in AKI
NGAL in AKINGAL in AKI
NGAL in AKI
 
Steroid Sparing Regimens in Kidney Transplantation
Steroid Sparing Regimens in Kidney TransplantationSteroid Sparing Regimens in Kidney Transplantation
Steroid Sparing Regimens in Kidney Transplantation
 
Aki biomarker 2016 chaken maniyan
Aki biomarker 2016 chaken maniyanAki biomarker 2016 chaken maniyan
Aki biomarker 2016 chaken maniyan
 
Salon b 13 kasim 15.45 17.00 müge aydoğdu-ing
Salon b 13 kasim 15.45 17.00 müge aydoğdu-ingSalon b 13 kasim 15.45 17.00 müge aydoğdu-ing
Salon b 13 kasim 15.45 17.00 müge aydoğdu-ing
 
Critical care nephrology
Critical care nephrologyCritical care nephrology
Critical care nephrology
 
When to initiate RRT in patients with AKI - Does timing matter?
When to initiate RRT in patients with AKI - Does timing matter?When to initiate RRT in patients with AKI - Does timing matter?
When to initiate RRT in patients with AKI - Does timing matter?
 
2016: Geriatric Nephrology - Beben
2016: Geriatric Nephrology - Beben2016: Geriatric Nephrology - Beben
2016: Geriatric Nephrology - Beben
 

Similar to Management of acute kidney injury

Aki an overview
Aki an overviewAki an overview
Aki an overviewFAARRAG
 
Acute kidney injury slideshare
Acute kidney injury slideshareAcute kidney injury slideshare
Acute kidney injury slideshareAzilah Sulaiman
 
Definition and staging criteria of acute kidney injury in adults up todate
Definition and staging criteria of acute kidney injury in adults up todateDefinition and staging criteria of acute kidney injury in adults up todate
Definition and staging criteria of acute kidney injury in adults up todateCamilaEscobar83
 
Acute Kidney Injury
Acute Kidney InjuryAcute Kidney Injury
Acute Kidney InjuryAbhijit Nair
 
When to Initiate RRT in Patients with AKI - Does Timing Matter?
When to Initiate RRT in Patients with AKI - Does Timing Matter?When to Initiate RRT in Patients with AKI - Does Timing Matter?
When to Initiate RRT in Patients with AKI - Does Timing Matter?Apollo Hospitals
 
AKI- Pharmacotherapy Handbook 2021 .pdf
AKI- Pharmacotherapy Handbook 2021 .pdfAKI- Pharmacotherapy Handbook 2021 .pdf
AKI- Pharmacotherapy Handbook 2021 .pdfjadarc
 
Aki dr osama el shahat 2017
Aki  dr osama el shahat  2017Aki  dr osama el shahat  2017
Aki dr osama el shahat 2017FarragBahbah
 
uremia, treatment, symptoms , 12222.pptx
uremia, treatment, symptoms , 12222.pptxuremia, treatment, symptoms , 12222.pptx
uremia, treatment, symptoms , 12222.pptxddjumanalieva97
 
When to Initiate RRT in Patients with AKI - Does Timing Matter?
When to Initiate RRT in Patients with AKI - Does Timing Matter?When to Initiate RRT in Patients with AKI - Does Timing Matter?
When to Initiate RRT in Patients with AKI - Does Timing Matter?Apollo Hospitals
 
cardiorenal syndrome and its characteristics and complications and causes.pptx
cardiorenal syndrome and its characteristics and complications and causes.pptxcardiorenal syndrome and its characteristics and complications and causes.pptx
cardiorenal syndrome and its characteristics and complications and causes.pptxArunDeva8
 
Think Kidneys and primary care
Think Kidneys and primary care Think Kidneys and primary care
Think Kidneys and primary care Renal Association
 
Approach To CKD Patient....................pptx.pptx
Approach To CKD Patient....................pptx.pptxApproach To CKD Patient....................pptx.pptx
Approach To CKD Patient....................pptx.pptximrulsujon1
 
AKIforResidentsInternalMedicine2022.pptx
AKIforResidentsInternalMedicine2022.pptxAKIforResidentsInternalMedicine2022.pptx
AKIforResidentsInternalMedicine2022.pptxhcahoustonim
 
Acute kidney injury: Perioperative implications
Acute kidney injury: Perioperative implicationsAcute kidney injury: Perioperative implications
Acute kidney injury: Perioperative implicationsAbhijit Nair
 
Dr. osama-el-shahat-aki-dep
Dr. osama-el-shahat-aki-depDr. osama-el-shahat-aki-dep
Dr. osama-el-shahat-aki-depFarragBahbah
 
Renal impairment and anaesthesia
Renal impairment and anaesthesiaRenal impairment and anaesthesia
Renal impairment and anaesthesiaErrol Williamson
 

Similar to Management of acute kidney injury (20)

Aki an overview
Aki an overviewAki an overview
Aki an overview
 
Acute kidney injury slideshare
Acute kidney injury slideshareAcute kidney injury slideshare
Acute kidney injury slideshare
 
Acute kidney injury
Acute kidney injury Acute kidney injury
Acute kidney injury
 
Aki icu
Aki icuAki icu
Aki icu
 
Definition and staging criteria of acute kidney injury in adults up todate
Definition and staging criteria of acute kidney injury in adults up todateDefinition and staging criteria of acute kidney injury in adults up todate
Definition and staging criteria of acute kidney injury in adults up todate
 
Acute Kidney Injury
Acute Kidney InjuryAcute Kidney Injury
Acute Kidney Injury
 
When to Initiate RRT in Patients with AKI - Does Timing Matter?
When to Initiate RRT in Patients with AKI - Does Timing Matter?When to Initiate RRT in Patients with AKI - Does Timing Matter?
When to Initiate RRT in Patients with AKI - Does Timing Matter?
 
International Journal of Hepatology & Gastroenterology
International Journal of Hepatology & GastroenterologyInternational Journal of Hepatology & Gastroenterology
International Journal of Hepatology & Gastroenterology
 
AKI- Pharmacotherapy Handbook 2021 .pdf
AKI- Pharmacotherapy Handbook 2021 .pdfAKI- Pharmacotherapy Handbook 2021 .pdf
AKI- Pharmacotherapy Handbook 2021 .pdf
 
Aki dr osama el shahat 2017
Aki  dr osama el shahat  2017Aki  dr osama el shahat  2017
Aki dr osama el shahat 2017
 
uremia, treatment, symptoms , 12222.pptx
uremia, treatment, symptoms , 12222.pptxuremia, treatment, symptoms , 12222.pptx
uremia, treatment, symptoms , 12222.pptx
 
When to Initiate RRT in Patients with AKI - Does Timing Matter?
When to Initiate RRT in Patients with AKI - Does Timing Matter?When to Initiate RRT in Patients with AKI - Does Timing Matter?
When to Initiate RRT in Patients with AKI - Does Timing Matter?
 
cardiorenal syndrome and its characteristics and complications and causes.pptx
cardiorenal syndrome and its characteristics and complications and causes.pptxcardiorenal syndrome and its characteristics and complications and causes.pptx
cardiorenal syndrome and its characteristics and complications and causes.pptx
 
Think Kidneys and primary care
Think Kidneys and primary care Think Kidneys and primary care
Think Kidneys and primary care
 
Approach To CKD Patient....................pptx.pptx
Approach To CKD Patient....................pptx.pptxApproach To CKD Patient....................pptx.pptx
Approach To CKD Patient....................pptx.pptx
 
AKIforResidentsInternalMedicine2022.pptx
AKIforResidentsInternalMedicine2022.pptxAKIforResidentsInternalMedicine2022.pptx
AKIforResidentsInternalMedicine2022.pptx
 
Acute kidney injury: Perioperative implications
Acute kidney injury: Perioperative implicationsAcute kidney injury: Perioperative implications
Acute kidney injury: Perioperative implications
 
Ngal lra
Ngal lraNgal lra
Ngal lra
 
Dr. osama-el-shahat-aki-dep
Dr. osama-el-shahat-aki-depDr. osama-el-shahat-aki-dep
Dr. osama-el-shahat-aki-dep
 
Renal impairment and anaesthesia
Renal impairment and anaesthesiaRenal impairment and anaesthesia
Renal impairment and anaesthesia
 

Recently uploaded

Recombinant DNA technology( Transgenic plant and animal)
Recombinant DNA technology( Transgenic plant and animal)Recombinant DNA technology( Transgenic plant and animal)
Recombinant DNA technology( Transgenic plant and animal)DHURKADEVIBASKAR
 
TOPIC 8 Temperature and Heat.pdf physics
TOPIC 8 Temperature and Heat.pdf physicsTOPIC 8 Temperature and Heat.pdf physics
TOPIC 8 Temperature and Heat.pdf physicsssuserddc89b
 
Bentham & Hooker's Classification. along with the merits and demerits of the ...
Bentham & Hooker's Classification. along with the merits and demerits of the ...Bentham & Hooker's Classification. along with the merits and demerits of the ...
Bentham & Hooker's Classification. along with the merits and demerits of the ...Nistarini College, Purulia (W.B) India
 
Call Girls in Hauz Khas Delhi 💯Call Us 🔝9953322196🔝 💯Escort.
Call Girls in Hauz Khas Delhi 💯Call Us 🔝9953322196🔝 💯Escort.Call Girls in Hauz Khas Delhi 💯Call Us 🔝9953322196🔝 💯Escort.
Call Girls in Hauz Khas Delhi 💯Call Us 🔝9953322196🔝 💯Escort.aasikanpl
 
Analytical Profile of Coleus Forskohlii | Forskolin .pdf
Analytical Profile of Coleus Forskohlii | Forskolin .pdfAnalytical Profile of Coleus Forskohlii | Forskolin .pdf
Analytical Profile of Coleus Forskohlii | Forskolin .pdfSwapnil Therkar
 
Manassas R - Parkside Middle School 🌎🏫
Manassas R - Parkside Middle School 🌎🏫Manassas R - Parkside Middle School 🌎🏫
Manassas R - Parkside Middle School 🌎🏫qfactory1
 
Evidences of Evolution General Biology 2
Evidences of Evolution General Biology 2Evidences of Evolution General Biology 2
Evidences of Evolution General Biology 2John Carlo Rollon
 
THE ROLE OF PHARMACOGNOSY IN TRADITIONAL AND MODERN SYSTEM OF MEDICINE.pptx
THE ROLE OF PHARMACOGNOSY IN TRADITIONAL AND MODERN SYSTEM OF MEDICINE.pptxTHE ROLE OF PHARMACOGNOSY IN TRADITIONAL AND MODERN SYSTEM OF MEDICINE.pptx
THE ROLE OF PHARMACOGNOSY IN TRADITIONAL AND MODERN SYSTEM OF MEDICINE.pptxNandakishor Bhaurao Deshmukh
 
Is RISC-V ready for HPC workload? Maybe?
Is RISC-V ready for HPC workload? Maybe?Is RISC-V ready for HPC workload? Maybe?
Is RISC-V ready for HPC workload? Maybe?Patrick Diehl
 
Call Girls in Munirka Delhi 💯Call Us 🔝8264348440🔝
Call Girls in Munirka Delhi 💯Call Us 🔝8264348440🔝Call Girls in Munirka Delhi 💯Call Us 🔝8264348440🔝
Call Girls in Munirka Delhi 💯Call Us 🔝8264348440🔝soniya singh
 
Grafana in space: Monitoring Japan's SLIM moon lander in real time
Grafana in space: Monitoring Japan's SLIM moon lander  in real timeGrafana in space: Monitoring Japan's SLIM moon lander  in real time
Grafana in space: Monitoring Japan's SLIM moon lander in real timeSatoshi NAKAHIRA
 
Temporomandibular joint Muscles of Mastication
Temporomandibular joint Muscles of MasticationTemporomandibular joint Muscles of Mastication
Temporomandibular joint Muscles of Masticationvidulajaib
 
Call Girls in Munirka Delhi 💯Call Us 🔝9953322196🔝 💯Escort.
Call Girls in Munirka Delhi 💯Call Us 🔝9953322196🔝 💯Escort.Call Girls in Munirka Delhi 💯Call Us 🔝9953322196🔝 💯Escort.
Call Girls in Munirka Delhi 💯Call Us 🔝9953322196🔝 💯Escort.aasikanpl
 
zoogeography of pakistan.pptx fauna of Pakistan
zoogeography of pakistan.pptx fauna of Pakistanzoogeography of pakistan.pptx fauna of Pakistan
zoogeography of pakistan.pptx fauna of Pakistanzohaibmir069
 
insect anatomy and insect body wall and their physiology
insect anatomy and insect body wall and their  physiologyinsect anatomy and insect body wall and their  physiology
insect anatomy and insect body wall and their physiologyDrAnita Sharma
 
Welcome to GFDL for Take Your Child To Work Day
Welcome to GFDL for Take Your Child To Work DayWelcome to GFDL for Take Your Child To Work Day
Welcome to GFDL for Take Your Child To Work DayZachary Labe
 
SOLUBLE PATTERN RECOGNITION RECEPTORS.pptx
SOLUBLE PATTERN RECOGNITION RECEPTORS.pptxSOLUBLE PATTERN RECOGNITION RECEPTORS.pptx
SOLUBLE PATTERN RECOGNITION RECEPTORS.pptxkessiyaTpeter
 
Artificial Intelligence In Microbiology by Dr. Prince C P
Artificial Intelligence In Microbiology by Dr. Prince C PArtificial Intelligence In Microbiology by Dr. Prince C P
Artificial Intelligence In Microbiology by Dr. Prince C PPRINCE C P
 
Call Us ≽ 9953322196 ≼ Call Girls In Mukherjee Nagar(Delhi) |
Call Us ≽ 9953322196 ≼ Call Girls In Mukherjee Nagar(Delhi) |Call Us ≽ 9953322196 ≼ Call Girls In Mukherjee Nagar(Delhi) |
Call Us ≽ 9953322196 ≼ Call Girls In Mukherjee Nagar(Delhi) |aasikanpl
 
Call Girls in Mayapuri Delhi 💯Call Us 🔝9953322196🔝 💯Escort.
Call Girls in Mayapuri Delhi 💯Call Us 🔝9953322196🔝 💯Escort.Call Girls in Mayapuri Delhi 💯Call Us 🔝9953322196🔝 💯Escort.
Call Girls in Mayapuri Delhi 💯Call Us 🔝9953322196🔝 💯Escort.aasikanpl
 

Recently uploaded (20)

Recombinant DNA technology( Transgenic plant and animal)
Recombinant DNA technology( Transgenic plant and animal)Recombinant DNA technology( Transgenic plant and animal)
Recombinant DNA technology( Transgenic plant and animal)
 
TOPIC 8 Temperature and Heat.pdf physics
TOPIC 8 Temperature and Heat.pdf physicsTOPIC 8 Temperature and Heat.pdf physics
TOPIC 8 Temperature and Heat.pdf physics
 
Bentham & Hooker's Classification. along with the merits and demerits of the ...
Bentham & Hooker's Classification. along with the merits and demerits of the ...Bentham & Hooker's Classification. along with the merits and demerits of the ...
Bentham & Hooker's Classification. along with the merits and demerits of the ...
 
Call Girls in Hauz Khas Delhi 💯Call Us 🔝9953322196🔝 💯Escort.
Call Girls in Hauz Khas Delhi 💯Call Us 🔝9953322196🔝 💯Escort.Call Girls in Hauz Khas Delhi 💯Call Us 🔝9953322196🔝 💯Escort.
Call Girls in Hauz Khas Delhi 💯Call Us 🔝9953322196🔝 💯Escort.
 
Analytical Profile of Coleus Forskohlii | Forskolin .pdf
Analytical Profile of Coleus Forskohlii | Forskolin .pdfAnalytical Profile of Coleus Forskohlii | Forskolin .pdf
Analytical Profile of Coleus Forskohlii | Forskolin .pdf
 
Manassas R - Parkside Middle School 🌎🏫
Manassas R - Parkside Middle School 🌎🏫Manassas R - Parkside Middle School 🌎🏫
Manassas R - Parkside Middle School 🌎🏫
 
Evidences of Evolution General Biology 2
Evidences of Evolution General Biology 2Evidences of Evolution General Biology 2
Evidences of Evolution General Biology 2
 
THE ROLE OF PHARMACOGNOSY IN TRADITIONAL AND MODERN SYSTEM OF MEDICINE.pptx
THE ROLE OF PHARMACOGNOSY IN TRADITIONAL AND MODERN SYSTEM OF MEDICINE.pptxTHE ROLE OF PHARMACOGNOSY IN TRADITIONAL AND MODERN SYSTEM OF MEDICINE.pptx
THE ROLE OF PHARMACOGNOSY IN TRADITIONAL AND MODERN SYSTEM OF MEDICINE.pptx
 
Is RISC-V ready for HPC workload? Maybe?
Is RISC-V ready for HPC workload? Maybe?Is RISC-V ready for HPC workload? Maybe?
Is RISC-V ready for HPC workload? Maybe?
 
Call Girls in Munirka Delhi 💯Call Us 🔝8264348440🔝
Call Girls in Munirka Delhi 💯Call Us 🔝8264348440🔝Call Girls in Munirka Delhi 💯Call Us 🔝8264348440🔝
Call Girls in Munirka Delhi 💯Call Us 🔝8264348440🔝
 
Grafana in space: Monitoring Japan's SLIM moon lander in real time
Grafana in space: Monitoring Japan's SLIM moon lander  in real timeGrafana in space: Monitoring Japan's SLIM moon lander  in real time
Grafana in space: Monitoring Japan's SLIM moon lander in real time
 
Temporomandibular joint Muscles of Mastication
Temporomandibular joint Muscles of MasticationTemporomandibular joint Muscles of Mastication
Temporomandibular joint Muscles of Mastication
 
Call Girls in Munirka Delhi 💯Call Us 🔝9953322196🔝 💯Escort.
Call Girls in Munirka Delhi 💯Call Us 🔝9953322196🔝 💯Escort.Call Girls in Munirka Delhi 💯Call Us 🔝9953322196🔝 💯Escort.
Call Girls in Munirka Delhi 💯Call Us 🔝9953322196🔝 💯Escort.
 
zoogeography of pakistan.pptx fauna of Pakistan
zoogeography of pakistan.pptx fauna of Pakistanzoogeography of pakistan.pptx fauna of Pakistan
zoogeography of pakistan.pptx fauna of Pakistan
 
insect anatomy and insect body wall and their physiology
insect anatomy and insect body wall and their  physiologyinsect anatomy and insect body wall and their  physiology
insect anatomy and insect body wall and their physiology
 
Welcome to GFDL for Take Your Child To Work Day
Welcome to GFDL for Take Your Child To Work DayWelcome to GFDL for Take Your Child To Work Day
Welcome to GFDL for Take Your Child To Work Day
 
SOLUBLE PATTERN RECOGNITION RECEPTORS.pptx
SOLUBLE PATTERN RECOGNITION RECEPTORS.pptxSOLUBLE PATTERN RECOGNITION RECEPTORS.pptx
SOLUBLE PATTERN RECOGNITION RECEPTORS.pptx
 
Artificial Intelligence In Microbiology by Dr. Prince C P
Artificial Intelligence In Microbiology by Dr. Prince C PArtificial Intelligence In Microbiology by Dr. Prince C P
Artificial Intelligence In Microbiology by Dr. Prince C P
 
Call Us ≽ 9953322196 ≼ Call Girls In Mukherjee Nagar(Delhi) |
Call Us ≽ 9953322196 ≼ Call Girls In Mukherjee Nagar(Delhi) |Call Us ≽ 9953322196 ≼ Call Girls In Mukherjee Nagar(Delhi) |
Call Us ≽ 9953322196 ≼ Call Girls In Mukherjee Nagar(Delhi) |
 
Call Girls in Mayapuri Delhi 💯Call Us 🔝9953322196🔝 💯Escort.
Call Girls in Mayapuri Delhi 💯Call Us 🔝9953322196🔝 💯Escort.Call Girls in Mayapuri Delhi 💯Call Us 🔝9953322196🔝 💯Escort.
Call Girls in Mayapuri Delhi 💯Call Us 🔝9953322196🔝 💯Escort.
 

Management of acute kidney injury

  • 1. Management of Acute Kidney Injury: Core Curriculum 2018 Peter K. Moore, Raymond K. Hsu, and Kathleen D. Liu Acute kidney injury (AKI) is a heterogeneous disorder that is common in hospitalized patients and associated with short- and long-term morbidity and mortality. When AKI is present, prompt workup of the underlying cause should be pursued, with specific attention to reversible causes. Measures to prevent AKI include optimization of volume status and avoidance of nephrotoxic medications. Crystalloids are preferred over colloids for most patients, and hydroxyethyl starches should be avoided. Volume overload in the setting of AKI is associated with adverse outcomes, so attention should be paid to overall fluid balance. Currently there are no targeted pharmacotherapies approved for the treatment of AKI. The optimal timing of renal replacement therapy in critically ill patients with AKI is unclear, but is an area of active investigation. Recent studies suggest that AKI is not a “self-limited” process, but is strongly linked to increased risk for chronic kidney disease, subsequent AKI, and future mortality. Introduction Acute kidney injury (AKI) is a common diagnosis in hospitalized patients, often occurring in patients with multiple comorbid conditions. It is associated with significant increases in both short- and long-term morbidity and mortality. Management of AKI requires an in-depth understanding of fluid and electrolyte homeostasis, as well as appropriate use of renal replacement therapy (RRT) in the acute setting. In this Core Cur- riculum, we review key principles regarding the diagnosis and general management of AKI for clinicians. Specific management of glomerulonephritis, thrombotic micro- angiopathies, and AKI in the setting of malignancy have been discussed in detail in previous Core Curriculum articles and are not addressed in depth. Diagnosis and Evaluation of AKI AKI is characterized by the accumulation of creatinine, urea, and other unmeasured waste products after an abrupt decrease in kidney function. More recently, injury has replaced “failure” to emphasize the disease continuum because even modest reductions in kidney function are associated with worse outcomes. AKI is common, affecting approximately 5% to 10% of hospitalized patients and up to 60% of patients admitted to the intensive care unit (ICU). AKI has a variety of causes and should not be viewed as a single disease. Although some management strategies can be universally applied, others are disease specific. Thus, when AKI is recognized, a critical first step is the prompt evaluation for the cause of AKI, with special attention to reversible causes. Definition Due to significant heterogeneity in prior studies, during the past 10 years efforts have been made to develop consensus AKI definitions, in particular for use in epidemi- ologic studies and clinical trials. In 2012, the KDIGO (Kidney Disease: Improving Global Outcomes) group combined elements from prior definitions (Table 1). Staging is based on both urine output and serum creatinine (Scr) concentration, and if urine output and Scr concentration do not correspond to the same stage, it has been recommended that the highest stage should be considered. However, although urine output is an important kidney function parameter that identifies patients at higher risk for adverse outcomes, its pathophysiologic significance in the absence of extremes of oliguria or other surrogates of reduced glomerular filtration rate (GFR) is more controversial. It should be noted that patients who develop AKI by KDIGO urine output criteria, regardless of whether Scr criteria are present, are at risk for developing fluid overload given the typically high obligate intake of critically ill patients. FEATURE EDITOR: Asghar Rastegar ADVISORY BOARD: Ursula C. Brewster Michael Choi Ann O’Hare Manoocher Soleimani The Core Curriculum aims to give trainees in nephrology a strong knowledge base in core topics in the specialty by providing an over- view of the topic and citing key references, including the founda- tional literature that led to current clinical approaches. Complete author and article information provided before references. Am J Kidney Dis. 72(1): 136-148. Published online February 22, 2018. doi: 10.1053/ j.ajkd.2017.11.021 © 2018 by the National Kidney Foundation, Inc. 136 AJKD Vol 72 | Iss 1 | July 2018 Core Curriculum
  • 2. Evaluation of Kidney Function in the Acute Care Setting At present, GFR is the gold-standard marker for acute or chronic kidney disease, though it represents only one of many affected functions. However, GFR is almost never directly measured in the clinical setting, and sur- rogate markers of kidney function are typically used. Current eGFR equations (Cockcroft-Gault, MDRD Study, and CKD-EPI) cannot be used when creatinine concentration is not at steady state, as occurs during AKI. Equations have been proposed to estimate kinetic GFR when Scr concentration is actively changing, but have not been validated for widespread use. In severe AKI (eg, when the patient is oligoanuric), the assumption should be that GFR is <10 mL/min when urine output is minimal. Furthermore, because Scr concentration lags acute changes in kidney function, the current AKI stage may not reflect cur- rent kidney function. Reductions in creatinine production during acute illness and sarcopenia (which often develops with prolonged illness), along with creatinine dilution during vol- ume overload, further complicate the evaluation of kidney function. Cystatin C has been used for GFR estimation and is thought to be more accurate at higher GFRs and in those with reduced muscle mass. However, the same limitations regarding steady-state kinetics apply, and the impact of volume of distribution has not been studied. There is consid- erable interest in developing bedside tools for real-time measuredGFR,butnosuchtoolsforclinicaluseexistatpresent. Tubular injury biomarkers include neutrophil gelatinase- associated lipocalin (NGAL), kidney injury molecule 1 (KIM- 1), interleukin 18 (IL-18), and liver-type fatty acid binding protein (L-FABP). Future definitions of AKI may incorporate biomarkers. There is also interest in biomarkers that reflect kidney stress, including tissue injury metalloproteinase 2 (TIMP-2) and insulin-like growth factor binding protein 7 (IGFBP-7), which have recently been approved by the US Food and DrugAdministration (FDA)fortheidentification of patients at high risk for developing KDIGO stage 2 to 3 AKI during the next 12 to 24 hours (these biomarkers are mar- keted as the Nephrocheck Test [Astute Medical]). Our un- derstanding of the clinical utility of this test is evolving rapidly; at present, this test may be useful to identify patients for implementation of care bundles (see below). Additional Readings ► KIDGO AKI Workgroup. KDIGO clinical practice guideline for AKI. Kidney Int Suppl. 2012;2:1-138. + ESSENTIAL READING Table 1. Comparison of Recent Consensus AKI Definitions AKI Stage Urine Outputa KDIGO AKIN RIFLE 1 <0.5 mL/kg/h for 6-12 h Scr to 1.5-1.9 × baseline over 7 d or ≥0.3 mg/dL absolute increase over 48 h Scr to 1.5-2 × baseline or ≥0.3 mg/dL absolute Scr increase within 48 h Risk: Scr to ≥1.5 × increase within 7 d, sustained for ≥24 h 2 <0.5 mL/kg/h for ≥12 h Scr to 2.0-2.9 × baseline Scr to >2-3 × baseline Injury: Scr to ≥2 × increase 3 <0.3 mL/kg/h for ≥24 h or anuria for ≥12 h Scr to ≥3.0 × baseline, or Scr increase to ≥4.0 mg/dL or initiation of RRT Scr to >3.0 × baseline, or Scr increase to ≥4.0 mg/dL (with increase of 0.5 mg/dL) or initiation of RRT Failure: Scr to ≥3.0 × increase or Scr increase to ≥4.0 mg/dL (with increase of 0.5 mg/dL) or initiation of RRT Loss: Complete loss of kidney function for >4 wk ESKD: ESKD for >3 mo Note: The first classification system, RIFLE, from the ADQI, incorporated 3 categories of injury and 2 outcomes that varied by severity. The outcomes (Loss, ESKD) were eliminated from the subsequent AKIN and KDIGO definitions. The AKIN definition incorporated smaller changes in Scr concentration, and the KDIGO definition added more definitive time frames to the definition. A key concept for the Scr-based definitions of AKI is the identification of baseline Scr concentration. Although the initial RIFLE criteria recommended the use of an Scr concentration that would equate to eGFR of 75 mL/min/1.73 m2 by the MDRD Study equation (MDRD-75) if no baseline was available, this definition does not account for chronic kidney disease if present. It is essential to look for a prior baseline/reference Scr concentration, ideally from the 365 days before hospital admission from a clinical context in which there was not concern for AKI (eg, a stable clinic visit). This concept is discussed in detail in the KDIGO AKI clinical practice guideline. Abbreviations: ADQI, Acute Dialysis Quality Initiative; AKI, acute kidney injury; AKIN, Acute Kidney Injury Network; eGFR, estimated glomerular filtration rate; ESKD, end-stage kidney disease; KDIGO, Kidney Disease: Improving Global Outcomes; MDRD, Modification in Diet in Renal Disease; RIFLE, risk, injury, failure, loss of kidney function, and end-stage kidney disease; RRT, renal replacement therapy; Scr, serum creatinine. a All 3 definitions (KDIGO, AKIN, RIFLE) use common urine output criteria. Case: You are asked to see a 50-year-old African American man with diabetes and known chronic kidney disease (CKD; baseline Scr, 2.0 mg/dL) who is admitted with urosepsis. Admission Scr concentration was 2.3 mg/dL and increased to 2.6 mg/dL the following day. Question 1: Which statement regarding his kidney function is correct? a) Using the MDRD (Modification of Diet in Renal Disease) Study equation, his estimated GFR (eGFR) is 34 mL/min/ 1.73 m2 b) Use of the CKD-EPI (CKD Epidemiology Collaboration) equation is more appropriate for this patient, and his eGFR is 32 mL/min/1.73 m2 c) Using the Cockcroft-Gault formula, his creatinine clear- ance is 20 to 32 mL/min d) His eGFR cannot be calculated because his Scr concentration is not stable For answer, see Appendix. Core Curriculum AJKD Vol 72 | Iss 1 | July 2018 137
  • 3. ► Kellum JA, Sileanu FE, Murugan R, Lucko N, Shaw AD, Clermont G. Classifying AKI by urine output versus serum creatinine level. J Am Soc Nephrol. 2015;26:2231-2238. ► Liu KD, Thompson BT, Ancukiewicz M, et al. AKI in patients with acute lung injury: impact of fluid accumulation on classification of AKI and associated outcomes. Crit Care Med. 2011;39:2665- 2671. ► Vijayan A, Faubel S, Askenazi DJ, et al. Clinical use of the urine biomarker [TIMP-2] x [IGFBP7] for AKI risk assessment. Am J Kidney Dis. 2016;68:19-28. Cause and General Management of AKI Evaluation of Cause of AKI Those who meet criteria for AKI should have the cause investigated, with special attention to treatable causes (Table 2). Careful history taking, chart review, and phys- ical examination remain the fundamental tenets of the workup. For example, after cardiopulmonary bypass sur- gery, AKI may be related to the bypass itself, hypovolemia, postoperative cardiogenic or (rarely early on) septic shock, or cholesterol emboli. Careful evaluation of the temporal pattern of AKI relative to the surgery and other clinical events (hypotension and iodinated contrast exposure), as well as physical examination and laboratory findings (eg, livedo reticularis and peripheral eosinophilia suggestive of cholesterol emboli) are needed to differentiate these conditions. All patients with AKI need careful assessment of hemodynamic and volume status using vital signs and physical examination; critically ill patients, for example, those in shock, may benefit from more invasive hemody- namic monitoring (arterial line, central venous pressure, or cardiac output monitoring). Urinary indexes (fractional excretion of sodium and urea) may be helpful in diagnosing decreased kidney perfusion (aka, prerenal azotemia) if the patient is oligoanuric. However, the utility of these indexes tends to be more limited in critically ill adults, likely as a result of coexisting pre- and intrarenal disease. Urinary microscopy for renal tubular epithelial cells and granular casts may be helpful to make the concomitant diagnosis of acute tubular necrosis (ATN), which is the most common cause of AKI occurring in the hospital. However, ATN is a misnomer because renal biopsy speci- mens from patients with this clinical diagnosis tend to have little frank necrosis and have evidence of significant nonlethal cell injury. Thus, ATN is used clinically to describe a specific and severe form of AKI that occurs from a variety of causes (Box 1), rather than the pathology per se. Table 2. Causes of AKI Type Examples of Specific Causes Decreased kidney perfusion (“prerenal” states) Hypovolemia Increased losses (hemorrhage, burns, massive vomiting or diarrhea), poor oral intake Reduced cardiac output Heart failure, cardiac tamponade, massive pulmonary embolism Renal vasomodulation/shunting Medications (NSAID, ACEi/ARB, cyclosporine, iodinated contrast), hypercalcemia, hepatorenal syndrome, abdominal compartment syndrome Systemic vasodilation Sepsis, SIRS, hepatorenal syndrome Intrarenal causes Vascular Renal artery stenosis, arterial/venous cross-clamping Microvascular Thrombotic microangiopathies (TTP, HUS, aHUS, DIC, APS, malignant hypertension, scleroderma renal crisis, preeclampsia/HELLP syndrome, drug-induced), cholesterol emboli Glomerular Rapidly progressive (crescentic) GN: anti–glomerular basement membrane; immune complex diseases: IgA nephropathy, postinfectious, lupus, mixed cryoglobuminemia with MPGN; pauci- immune glomerulonephritis: ANCA-associated vasculitides: GPA, MPA, EGPA (Churg-Strauss); ANCA-negative; nephrotic-range proteinuria with associated AKI: HIV-associated nephropathy (secondary FSGS); other causes of nephrotic-range proteinuria that commonly associate with AKI: minimal change disease with ATN/AIN; membranous nephropathy + crescentic GN or renal vein thrombosis; myeloma + multiple different pathologies, but in particular light chain cast nephropathy Tubulointerstitium AIN: medications, infection, lymphoproliferative disease; pigment nephropathy: rhabdomyolysis (myoglobin), massive hemolysis (hemoglobin); crystal nephropathy: uric acid (tumor lysis), acyclovir, sulfonamides, protease inhibitors (indinavir, azatanavir), methotrexate, ethylene glycol, acute phosphate nephropathy, oxalate nephropathy; myeloma-associated AKI (cast nephropathy); ATN: ischemia (shock, sepsis), inflammatory (sepsis, burns), medications (see Box 1; osmotic nephrosis in setting of sucrose, mannitol and hydroxyethylstarch use) Postrenal causes Bladder outlet Benign prostatic hypertrophy, cancer, strictures, blood clots Ureteral Bilateral obstruction (or unilateral with one kidney): stones, malignancy, retroperitoneal fibrosis Renal pelvis Papillary necrosis (NSAIDs), stones Note: Causes of AKI can be broadly divided into prerenal, intrarenal, and postrenal causes and then further subdivided as described. Abbreviations: ACEi, angiotensin-converting enzyme inhibitor; (a)HUS, (atypical) hemolytic uremic syndrome; AKI, acute kidney injury; ANCA, antineutrophil cytoplasmic antibody; APS, antiphospholipid syndrome; ATN/AIN, acute tubular necrosis/acute interstitial nephritis; ARB, angiotensin receptor blocker; DIC, disseminated intravas- cular coagulation; EGPA, eosinophilic granulomatosis with polyangiitis; FSGS, focal segmental glomerulosclerosis; GN, glomerulonephritis; GPA, granulomatosis with polyangiitis; HELLP, hemolysis, elevated liver enzymes, low platelet count) syndrome; HIV, human immunodeficiency virus; MPA, microscopic polyangiitis; MPGN, membranoproliferative glomerulonephritis; NSAID, nonsteroidal anti-inflammatory drug; SIRS, systemic inflammatory response syndrome; TTP, thrombotic thrombocy- topenic purpura. 138 AJKD Vol 72 | Iss 1 | July 2018 Core Curriculum
  • 4. Patients suspected of having a specific treatable intra- renal cause of AKI (such as acute interstitial nephritis [AIN], glomerulonephritis, or thrombotic micro- angiopathy) should have a urine sediment examination and serologic/hematologic tests, as indicated. A kidney biopsy should be considered when there is significant new proteinuria (protein excretion > 3 g/d) or hematuria, active urine sediment, or no readily identifiable cause of decreased kidney perfusion, obstruction, or ATN. A retrospective study of 68 critically ill patients who un- derwent kidney biopsy based on clinical suspicion found that 51% of patients had a specific cause of AKI, which led to a significant change in treatment plan in 21%. However, kidney biopsy was associated with complications in 22%, most commonly from bleeding. AIN is likely underdiagnosed in hospitalized patients who develop AKI. The widespread use of antibiotics and proton pump inhibitors puts these patients at higher risk for AIN. When not accompanied by systemic symp- toms (eg, rash and eosinophilia), AIN can be difficult to diagnose. Urinary eosinophils have been demonstrated to have poor test characteristics, and kidney biopsy is the only definitive way to establish the diagnosis. Treatment of AIN involves cessation of the culprit medication (if drug induced) and consideration of steroid therapy. Renal ultrasonography or computed tomography of the abdomen and pelvis without iodinated contrast is indicated when obstruction is suspected. In individuals with 2 kidneys, obstruction must be bilateral to cause AKI. In those for whom there is another clear cause for AKI, routine imaging may not be warranted. Overview of AKI Management Patients at risk for AKI and those with AKI should have kidney function monitored closely by Scr concentration and urine output (Fig 1). Careful assessment of volume status and hemodynamics should be undertaken and treated with intravenous fluids, diuretics, or other means of hemodynamic support as indicated. These treatments, along with RRT, are discussed in subsequent sections. Medications should be reviewed closely for nephrotoxic agents, which should be discontinued or switched to med- ications with less nephrotoxic potential. In a quality improvement initiative, a pharmacy-led notification for pediatric patients receiving 3 or more nephrotoxic medi- cations or an aminoglycoside resulted in a 38% decrease in nephrotoxic medication exposure and 64% decrease in AKI incidence. In addition, medications that may accumulate with reduced GFR should be avoided or adjusted, in particular in patients with stage 2 or 3 AKI (Box 2). Although not a medication per se, in patients with AKI, exposure to gadolinium has been associated with nephrogenic systemic fibrosis, a sclerosing condition of the skin and internal organs that can result in death. Although the absolute inci- dence of this condition is low, the relative risks and benefits of gadolinium administration must be cautiously weighed. Newer gadolinium preparations may be associated with a lower risk of nephrogenic systemic fibrosis. With regard to specific nephrotoxins, there is growing interest in the nephrotoxic effects of vancomycin, which in the setting of higher target trough concentrations for severe methicillin-resistant Staphylococcus aureus (MRSA) infections and declining kidney function can accumulate to very high levels (>50 μg/mL). Casts that contain nanospheric vancomycin have recently been described in individuals with vancomycin-associated AKI. The addition of piperacillin/tazobactam may potentiate the nephrotoxicity of vancomycin, but the mechanism is unclear. Nonsteroidal anti-inflammatory drugs and angiotensin-converting enzyme inhibitors/angiotensin receptor blockers are common classes of medications that should be discontinued. Although recent studies suggest Box 1. Medications Commonly Associated With Acute Tubular Necrosis • Aminoglycosides (tobramycin, gentamycin) • NSAIDs (ibuprofen, naproxen, ketorolac, celecoxib) • ACEi (captopril, lisinopril, benazepril, ramipril) • ARB (losartan, valsartan, candesartan, irbesartan) • Amphotericin • Cisplatin • Foscarnet • Iodinated contrast • Pentamidine • Tenofovir • Zolendronic acid Note: Although not a classic cause of acute tubular necrosis, volume depletion caused by diuretics can exacerbate the effects of some of these other medica- tions. This table does not include common causes of pigment or crystal ne- phropathy (which are described in Table 2) or medications associated with osmotic injury. Abbreviations: ACEi, angiotensin-converting enzyme inhibitor; ARB, angiotensin receptor blocker; NSAIDs, nonsteroidal anti-inflammatory drugs. Box 2. Key Medications Requiring Dose Adjustment (or Cessation) in AKI • Analgesics (morphine, meperidine, gabapentin, pregabalin) • Antiepileptics (lamotrigine) • Antivirals (acyclovir, gancyclovir, valgancyclovir) • Antifungals (fluconazole) • Antimicrobials (almost all antimicrobials need dose adjust- ment in AKI, with important exceptions of azithromycin, cef- triaxone, doxycycline, linezolid, moxifloxacin, nafcillin, rifampin) • Diabetic agents (sulfonylureas, metformin) • Allopurinol • Baclofen • Colchicine • Digoxin • Lithium • Low-molecular-weight heparin • NOACs Note: Medications that are associated with acute tubular necrosis (Box 1) should be withheld, if possible. Abbreviations: AKI, acute kidney injury; NOAC, novel anticoagulants. AJKD Vol 72 | Iss 1 | July 2018 139 Core Curriculum
  • 5. that the association between iodinated radiocontrast and AKI may not be as strong as previously thought, iodinated contrast should be avoided if possible in patients with or at risk for AKI. A number of recent studies have examined bundled protocols to improve the quality and consistency of care for patients with or at risk for AKI. In one study, 276 patients undergoing cardiac surgery who had elevated TIMP-2 × IGFBP-7 concentrations were randomly assigned to routine care or a strictly implemented AKI prevention protocol (from the KDIGO guideline and consisting of items such as hemodynamic optimization and avoidance of nephrotoxins). Postoperative AKI was observed to be significantly lower in the protocol group (55% vs 72%; absolute risk reduction, 17%; P = 0.004). Notably, the biomarker strategy enriched for high-risk patients, reducing the number needed to treat. However, more work is needed to design and implement such potentially successful (and sustainable) care bundles for AKI prevention and management. Along the same lines, there has been tremendous interest in the use of electronic alert sys- tems to identify patients with early AKI or at high risk for AKI, but the effectiveness of these alerts to change clinical practice has been variable and limited to date. Additional Readings ► Augusto J-F, Lassalle V, Fillatre P, et al. Safety and diagnostic yield of renal biopsy in the intensive care unit. Intensive Care Med. 2012;38:1826-1833. ► Darmon M, Ostermann M, Cerda J, et al. Diagnostic work-up and specific causes of AKI. Intensive Care Med. 2017;43:829-840. + ESSENTIAL READING ► Goldstein SL, Mottes T, Simpson K, et al. A sustained quality improvement program reduces nephrotoxic medication- associated AKI. Kidney Int. 2016;90:212-221. ► Kolhe NV, Reilly T, Leung J, et al. A simple care bundle for use in AKI: a propensity score-matched cohort study. Nephrol Dial Transplant. 2016;31:1846-1854. ► Lachance P, Villeneuve PM, Rewa OG, et al. Association between E-alert implementation for detection of AKI and outcomes: a systematic review. Nephrol Dial Transplant. 2017;32:265-272. + ESSENTIAL READING ► Luque Y, Louis K, Jouanneau C, et al. Vancomycin-associated cast nephropathy. J Am Soc Nephrol. 2017;28:1723-1728. ► Meersch M, Schmidt C, Hoffmeier A, et al. Prevention of cardiac surgery-associated AKI by implementing the KDIGO guidelines in high risk patients identified by biomarkers: the PREVAKI randomized controlled trial. Intensive Care Med. 2017;43(11): 1551-1561. ► Moledina DG, Perazella MA. Drug-induced acute interstitial nephritis. Clin J Am Soc Nephrol. 2017;12(12):2046-2049. + ESSENTIAL READING ► Perazella MA, Coca SG, Kanbay M, Brewster UC, Parikh CR. Diagnostic value of urine microscopy for differential diagnosis of AKI in hospitalized patients. Clin J Am Soc Nephrol. 2008;3:1615-1619. Hemodynamic Support: Fluid Management and Blood Pressure Targets Management of hemodynamics in patients with AKI, especially those in shock, is of critical importance. Although under normal conditions relatively constant renal blood flow can be maintained despite changes in blood pressures through autoregulation, these mechanisms are disrupted in AKI. Titration of fluids and vasopressors can be complex: hypotension can result in continued kidney damage in those with AKI, whereas administration of va- sopressors in those without adequate intravascular volume can further reduce renal blood flow. Conversely, patients with AKI are at risk for volume overload, and intravenous fluid loading may cause harm. Intravenous Fluid Resuscitation Outside the setting of iodinated contrast administration, there are no randomized trials comparing intravenous fluids to placebo for AKI prevention. However, it can be assumed that those with reduced renal blood flow who can augment their cardiac output by expansion of their intra- vascular volume would benefit from fluid resuscitation. Early goal-directed therapy, in which septic patients received intravenous crystalloids, inotropes, and trans- fusions according to predefined protocols, had no effect on mortality or need for RRT in 3 subsequent large trials. Although administration of intravenous fluids in patients with sepsis and/or hypovolemia is beneficial initially, fluid overload, especially in later disease, may confer harm. Several retrospective studies have found associations between positive fluid balance and mortality in critically ill patients. In a large multicenter cohort focused on critically ill patients, those with fluid overload (10% weight gain) at the time of dialysis therapy initiation had an odds ratio (OR) for death of 2.07 (95% confidence interval [CI], 1.27-3.37); findings were similar in those with AKI who Case, continued: Review of the patient’s chart shows that he has received 4 L of 0.9% saline solution intravenously in the past 24 hours, and urine output has increased from 10 to 20 mL/h. On physical examination, vital signs include blood pressure of 95/65 mm Hg, heart rate of 72 beats/min, and oxygen saturation of 96% on 2 L/min by nasal cannula. His lungs are clear. He has peripheral edema (2+). Question 2: What would you recommend? a) Continue with volume expansion because his urine output has increased significantly b) Add norepinephrine to increase his systolic blood pres- sure to >105 mm Hg c) Continue with volume expansion and add norepinephrine as well d) Start a trial of intravenous furosemide, which could help manage his fluid overload For answer, see Appendix. 140 AJKD Vol 72 | Iss 1 | July 2018 Core Curriculum
  • 6. did not require dialysis. However, such analyses of fluid overload are likely partially confounded by severity of illness. At present, there are numerous methods that can be used to assess fluid responsiveness, and no one method can be recommended above others. We recommend using multiple clinical assessments and repeated measures to assess fluid responsiveness. Intravenous fluids should be used judiciously in patients with AKI who are not “volume responsive.” After significant volume resuscitation, even if patients remain volume responsive, vasopressor support should be considered to avoid markedly positive fluid balance. In those requiring volume resuscitation, the choice of solution is controversial. Major trials of various colloids, physiologic-balanced salt solutions, and saline solution have been completed. We next review the evi- dence base for fluid selection. Colloid Versus Crystalloid Colloids, such as albumin, hydroxyethyl starches (HESs), and gelatins, rely on oncotic gradients to selectively expand the intravascular space, while crystalloids equili- brate across intravascular and extravascular spaces. Patients with inflammatory states will have increased vascular permeability, and some of this benefit may be lost. Albumin appears to be a relatively safe, albeit more expensive, alternative for resuscitation of critically ill pa- tients. In the Saline Versus Albumin Fluid Evaluation (SAFE) trial, ICU patients who received 4% albumin had no renal or mortality benefit. However, less total volume was required for resuscitation in the albumin group (2.2 vs 3.1 L). Given the reduction in volume needed, albumin may have a role in special situations in which large vol- umes of intravenous fluids are anticipated, such as septic shock in a cirrhotic patient. There is a clear indication for albumin in the setting of large-volume paracentesis for patients with end-stage liver disease because albumin infusion is associated with lower risk for AKI. Albumin (and likely other colloids) should be avoided in patients with traumatic brain injury due to an increased risk for death. There are a variety of HES preparations with differing molecular weights, molar substitutions, and tonicities, all of which are relatively inexpensive compared to albumin. Several trials have demonstrated renal toxicity with hyperoncotic HES administration due to proximal tubule vacuolization and swelling (osmotic nephrosis). Subse- quently, trials of iso-oncotic HES preparations have tested the hypothesis that these preparations are less nephrotoxic. The Crystalloid Versus Hydroxyethyl Starch (CHEST) Study randomly assigned 7,000 ICU patients to receive saline solution or an iso-oncotic 6% HES and found an increased risk for RRT in the group that received HES (7.0% vs 5.3%; P = 0.04). This study demonstrates one of the potential challenges of the combined Scr concentration and urine output–based AKI criteria: although there was more AKI defined by RIFLE (risk, injury, failure, loss of kidney function, and end-stage kidney disease) risk or injury in the saline-solution arm, this was largely driven by urine output. In contrast, the HES group, which had a lower overall rate of AKI, had higher rates of RRT and a trend toward more severe AKI. Following the publication of this study, the FDA added additional warnings to the packaging for HES. The other synthetic colloids commonly used for volume expansion are gelatins, but there are substantially fewer data regarding the association of gelatins with AKI. In general, given the lack of clear benefit with colloid administration, routine use of these solutions is not warranted. Finally, there is still interest in the role of colloids for the treatment of hypovolemic shock. Colloids Versus Crystalloids for the Resuscitation of the Critically Ill (CRISTAL) was a multicenter randomized open-label study of more than 2,800 ICU patients with hypovolemic shock. Patients were randomly assigned to fluid type (crystalloid or colloid), and the selection of fluid was up to the study investigator. About 45% of those in the colloid arm received HES. There was no difference in RRT requirement or mortality at 28 days (primary study end point). How- ever, there was a significant reduction in mortality at 90 days, need for mechanical ventilation, and need for vaso- pressors in those who received colloids. Thus, it has been suggested that a key to the use of colloids is the optimal timing of administration. Regardless, at present, there are no data to support the routine use of colloid for volume resuscitation. Physiologic Balanced Salt Solution Versus Normal Saline Solution Isotonic 0.9% saline solution has a significantly higher chloride content than the extracellular space in humans (154 vs w110 mmol/L), and patients receiving normal saline solution are at risk for hyperchloremic metabolic acidosis. Hyperchloremia has been associated with increased renal vascular resistance, increased renin activity, and decreased GFR in animal studies. In healthy volunteers, administration of 0.9% saline solution is associated with increased extravascular volume and decreased renal cortical tissue perfusion compared to a balanced salt solution. To explore the hypothesis that chloride-rich fluids increase the risk for AKI, Yunos et al performed an open- labeled sequential study of ICU patients at a single institution. During a 6-month period, patients were administered balanced salt solutions for resuscitation and were compared with controls from the corresponding 6 months from 1 year prior. The chloride-restricted group had a lower incidence of AKI (8.4% vs 14%) and lower rates of RRT (6.3% vs 10%). Subsequently, the SPLIT (0.9% Saline vs Plasma-Lyte 148 for ICU Fluid Therapy) trial, a multicenter randomized double-blind crossover study, did not find a significant difference in rates of AKI, AJKD Vol 72 | Iss 1 | July 2018 141 Core Curriculum
  • 7. need for RRT, or mortality between the 0.9% saline so- lution versus Plasma-Lyte groups (Baxter). However, this study has been criticized because it was a predominantly postoperative population that received only modest resuscitation volumes (median, 2 L). Recently, data from large pragmatic trials focused on patients admitted to the emergency department or ICU at a single US institution suggests benefit with balanced salt administration with regard to the composite end point of Major Adverse Kid- ney Events to day 30, defined as death, need for RRT or persistently decreased kidney function at day 30/hospital discharge (ClinicalTrials.gov identifiers NCT02444988, NCT02547779, and NCT02614040). Blood Pressure Management There has been interest in optimal blood pressure tar- gets in patients with shock. The SEPSISPAM (Sepsis and Mean Arterial Pressure) trial randomly assigned patients with septic shock requiring vasopressors to 2 blood pressure goals, a standard mean arterial pressure (MAP) goal (65-70 mm Hg) and a higher goal (80-85 mm Hg). There was no difference in mortality between the 2 treatment groups. However, patients with chronic hypertension in the higher MAP group had significantly lower rates of AKI and RRT. The number needed to treat to prevent 1 patient with hypertension from needing RRT was modest, at 9.5. Patients in the higher MAP group had higher rates of atrial fibrillation. Thus, blood pressure targets should likely take into account pre- morbid blood pressures, weighing the potential benefits of increased renal perfusion against the potentially deleterious effects of vasoconstriction resulting in hypoperfusion of other organs. Additional Readings ► Annane D, Siami S, Jaber S, et al. Effects of fluid resuscitation with colloids vs crystalloids on mortality in critically ill patients presenting with hypovolemic shock: the CRISTAL randomized trial. JAMA. 2013;310:1809-1817. ► Asfar P, Meziani F, Hamel J-F, et al. High versus low blood- pressure target in patients with septic shock. N Engl J Med. 2014;370:1583-1593. ► Bouchard J, Soroko SB, Chertow GM, et al. Fluid accumulation, survival and recovery of kidney function in critically ill patients with AKI. Kidney Int. 2009;76:422-427. ► Chowdhury AH, Cox EF, Francis ST, Lobo DN. A randomized, controlled, double-blind crossover study on the effects of 2L in- fusions of 0.9% saline and Plasma-Lyte(R) 148 on renal blood flow velocity and renal cortical tissue perfusion in healthy volunteers. Ann Surg. 2012;256:18-24. ► Davison D, Junker C. Advances in critical care for the nephrolo- gist: hemodynamic monitoring and volume management. Clin J Am Soc Nephrol. 2008;3:554-561. + ESSENTIAL READING ► Finfer S1, Bellomo R, Boyce N, French J, Myburgh J, Norton R; SAFE Study Investigators. A comparison of albumin and saline for fluid resuscitation in the ICU. N Engl J Med. 2004;350:2247-2256. ► Myburgh JA, Finfer S, Bellomo R, et al. Hydroxyethyl starch or saline for fluid resuscitation in intensive care. N Engl J Med. 2012;367:1901-1911. ► Perner A, Prowle J, Joannidis M, Young P, Hjortrup PB, Pettil€a V. Fluid management in AKI. Intensive Care Med. 2017;43: 807-815. ► PRISM Investigators; Rowan KM, Angus DC, Bailey M, et al. Early, goal-directed therapy for septic shock — a patient-level meta-analysis. N Engl J Med. 2017;376:2223-2234. ► Semler MW, Self W, Rice TW. Balanced crystalloids vs saline for critically ill adults. Chest. 2017;152(4 suppl):A1120. ► Young P, Bailey M, Beasley R, et al. Effect of a buffered crystalloid solution vs saline on AKI among patients in the ICU: the SPLIT randomized clinical trial. JAMA. 2015;314: 1701-1710. ► Yunos NaM, Bellomo R, Hegarty C, Story D, Ho L, Bailey M. Association between a chloride-liberal vs chloride-restrictive intravenous fluid administration strategy and kidney injury in criti- cally ill adults. JAMA. 2012;308:1566-1572. Additional Therapies for AKI: Diuretics, Nutrition, and the Future Diuretics Loop diuretics are commonly used in oliguric AKI despite the lack of evidence for their benefit. In addition to pre- venting volume overload, loop diuretics theoretically attenuate ischemic tubular injury by decreasing metabolic demand in the oxygen-poor renal medulla by inhibition of the sodium/potassium/chloride (Na+ /K+ /2Cl- ) cotrans- porter. However, clinical trials have failed to consistently show a benefit of diuretics in AKI. Thus, KDIGO recom- mends against the use of diuretics to treat AKI except in the setting of volume overload, when they can be used for management of volume overload itself. It has been pro- posed that in early AKI, urine output response to loop diuretics may have prognostic value. The hypothesis is that patients with AKI who are able to augment urine output in response to a diuretic challenge have intact tubular func- tion and therefore may have a better renal prognosis. However, this finding has not been validated in large multicenter studies. Nutrition and Glucose Control AKI is a catabolic state, and patients with AKI may need enteral or parenteral nutritional support. In general, the enteral route is preferred due to the lower risk for infection (and lower volumes needed to administer equivalent calories). The nutrition prescription in AKI will vary significantly depending on the underlying cause of AKI and the form of RRT provided, if any. With regard to glycemic control, the KDIGO guideline recommends maintaining blood glucose concentrations between 110 and 149 mg/dL in critically ill patients, a range that has never been formally evaluated in randomized trials. The potential renal benefit of glucose control was demonstrated in a single-center study of surgical patients randomly assigned to a target blood glucose concentration of 80 to 110 mg/dL or 180 to 200 mg/dL, in which the incidence of severe AKI and RRT was lower in the intensive arm (4.8% vs 8.2%). A notable practice difference in this study was the provision of dextrose in the immediate 142 AJKD Vol 72 | Iss 1 | July 2018 Core Curriculum
  • 8. postoperative period, which may have increased the adverse consequences of hyperglycemia. Subsequently, the NICE-SUGAR (Normoglycemia in Intensive Care Evaluation–Survival Using Glucose Algorithm Regulation) Study, the largest randomized clinical trial of glycemic control in critically ill patients, highlighted the potential risks of intensive glycemic control. Participants (6,100 in total) were randomly assigned to intensive (81-108 mg/ dL) or conventional (<180 mg/dL) glycemic control. There was no difference in rates of RRT between groups. However, intensive glycemic control was associated with higher mortality (OR, 1.14; 95% CI, 1.02-1.28) and a greater incidence of severe hypoglycemia (6.8% vs 0.5%). Several additional clinical trials have had similar findings. Patients with AKI may be at particularly high risk for severe hypoglycemia given the kidney’s role in insulin meta- bolism and glucose excretion. However, severe hypergly- cemia is associated with increased morbidity and mortality in a variety of clinical scenarios and should also be avoided. Pharmacotherapies for AKI At this time, there are no pharmacologic therapies for the prevention or treatment of AKI (Box 3). Because AKI is a heterogeneous disease, identification of a single therapy that will benefit all is challenging. Additionally, the AKI insult almost always precedes AKI detection and it is therefore difficult to intervene before the disease is established. Early identification and treatment of AKI with drugs that have pleiotropic effects on multiple pathologic pathways are most likely to be successful. Additional Readings ► Chawla LS, Davison DL, Brasha-Mitchell E, et al. Development and standardization of a furosemide stress test to predict the severity of AKI. Crit Care. 2013;17:R207. ► NICE-SUGAR Study Investigators; Finfer S, Chittock DR, Su SY, et al. Intensive versus conventional glucose control in critically ill patients. N Engl J Med. 2009;360:1283-1297. ► Siew ED, Liu KD. Nutritional support in AKI. In: Mitch W, Ikizler TA, eds. Handbook of Nutrition and the Kidney. Philadelphia, PA: Wolters Kluwer; 2018. ► van den Berghe G, Wouters P, Weekers F, et al. Intensive insulin therapy in critically ill patients. N Engl J Med. 2001;345:1359- 1367. ► Vanmassenhove J, Kielstein J, J€orres A, Biesen WV. Management of patients at risk of AKI. Lancet. 2017;389:2139-2151. + ESSENTIAL READING Management of Severe AKI, Including RRT Patients with AKI may develop hyperkalemia, metabolic acidosis, volume overload, and/or symptoms of uremia due to reduced GFR. Hyperkalemia can be medically managed as described in Table 3. Metabolic acidosis may occur due to AKI itself (eg, inability to excrete organic acids) or conditions associated with AKI (eg, hypo- perfusion leading to lactic acidosis). The kidney plays an important role, along with the liver, in lactate metabolism. Treatment of metabolic acidosis depends on its severity and must take into consideration absolute pH, rate of change of acidosis, and its underlying cause. Metabolic acidosis itself can be treated with bicarbonate or other base equivalents. Diuretics can be used to manage volume overload. Despite these temporizing measures, some with severe AKI will require RRT. The optimal timing of RRT is an area of active investigation. Factors that may affect the timing of RRT initiation are reviewed in the Continuous Dialysis Therapies Core Curriculum. With regard to available data, retrospective studies have showed an association between early RRT and favorable outcomes. However, Box 3. Agents Tested in Selected Trials for Treatment of AKI Trials ongoing • Alkaline phosphatase (sepsis-associated AKI) • L-Carnitine (sepsis-associated AKI) • Remote ischemic preconditioning (post operative AKI) • p53-targeted siRNA (post–cardiac surgery AKI) • Extracorporeal devices (dialysis-requiring AKI) • Vitamin D (hospitalized AKI) • Uremic toxin absorption/pentoxifylline (hospital-acquired AKI) No clear evidence of benefit • α-Melanocyte-stimulating hormone • Atrial natriuretic peptide • Calcium channel blockers • Diureticsa • Dopamine • Erythropoietin • Fenoldopam • Insulin growth factor • N-Acetylcysteine • Statins • Aminophylline/theophyllineb Abbreviations: AKI, acute kidney injury; siRNA, short interfering RNA. a Potentially useful for volume management, but not for treatment of AKI. b Some interest remains for AKI prevention in neonates. Case, continued: Two days later, the patient remains oli- guric despite a trial of furosemide, and his Scr concentration has increased to 5.5 mg/dL. Blood pressure is 105/75 mm Hg. He has peripheral edema (3+), and oxygen saturation is 91% on 5 L/min by nasal cannula. You plan to initiate RRT. Question 3: Which of the following is the best state- ment with respect to this set of circumstances? a) Given the role of sepsis in development of his AKI, continuous RRT (CRRT) is preferable to intermittent he- modialysis (IHD) for this patient b) If CRRT is selected, the prescribed dose should be 35 to 40 mL/kg/h c) CRRT and IHD have similar clinical outcomes d) RRT can be postponed until the patient develops clear signs of uremia For answer, see Appendix. AJKD Vol 72 | Iss 1 | July 2018 143 Core Curriculum
  • 9. many of these studies were limited in their assessment of “early” based on serum urea nitrogen or creatinine con- centration without other clinical information. Recently, 2 trials have assessed the impact of RRT timing in ICU pa- tients. The ELAIN (Early Versus Late Initiation of Renal Replacement Therapy in Critically Ill Patients With Acute Kidney Injury) trial found that 90-day mortality was significantly lower in patients randomly assigned to earlier RRT. However, this study has been criticized as a single- center trial that included many post–cardiac surgery pa- tients and enrolled patients with early AKI (KDIGO stage 2 AKI and elevated plasma NGAL). In contrast, the AKIKI (Artificial Kidney Initiation in Kidney Injury) Study was a multicenter trial that randomly assigned patients with more severe (KDIGO stage 3) AKI and did not find a dif- ference in mortality between early and delayed RRT. Thus, the questions of whether early RRT is beneficial, and if so, in which patients, remain unanswered. Two large ongoing trials will help answer these questions (ClinicalTrials.gov identifiers NCT01682590 and NCT02568722). Interest- ingly, in the STARRT-AKI (Standard vs. Accelerated Initi- ation of RRT in Acute Kidney Injury) pilot and in AKIKI, a significant proportion of participants in the late initiation arm recovered kidney function before RRT. This finding suggests that in addition to patients who may benefit from the earlier provision of RRT, there are patients who may recover before the need for RRT, and our ability to identify these patients is limited at best. RRT Prescription, Including Modality and Dose Several aspects of the RRT prescription, including site selection for vascular access, choice of membrane and anti- coagulation, and differences between convective and diffusive clearance, are discussed in detail in the Continuous Dialysis Therapies Core Curriculum. With regard to modality, the most widely used are CRRT and IHD. Pro- longed intermittent RRT/sustained low-efficiency dialysis are additional options that are used less frequently. Although they appear to have similar outcomes in preliminary studies and meta-analyses comparing these modalities with other forms of RRT for AKI, there is a need for better quality evi- dence in these areas before their routine use can be recom- mended. Peritoneal dialysis can be used in the acute setting as well and can be of particular use in resource-limited settings. There has been much interest in whether CRRT is associated with more favorable outcomes, including lower mortality and enhanced renal recovery. However, to date, small randomized clinical trials and meta-analyses have found no association between modality and outcome (mortality or renal recovery). Thus, as recommended by the KDIGO guideline, CRRT and IHD are complementary therapies; treatment considerations include the individual patient’s hemodynamic status, degree of volume overload and bleeding risk, and the treating facility’s availability/ experience. An early single-center trial suggested that patients with higher CRRT intensity (35 or 45 mL/kg/h) had lower mortality when compared to lower intensity (20 mL/kg/ h). However, 2 subsequent multicenter randomized controlled trials, the VA/NIH ATN (Veterans Affairs/Na- tional Institutes of Health Acute Renal Failure Trial Network) and ANZICS RENAL (Australian and New Zea- land Intensive Care Society Randomised Evaluation of Normal Versus Augmented Level of Renal Replacement Therapy in ICU) studies, found that there was no differ- ence in mortality or renal recovery when comparing high- to low-intensity RRT. Patients in the high-intensity arm were more likely to have hypophosphatemia. Table 3. Medical Management of Hyperkalemia Purpose of Treatment Drug Usual Dose Notes Stabilization of cardiac membrane Calcium gluconate or calcium chloride 1 g IV over minutes, repeat as needed Given when ECG changesa present; use with caution with digoxinb Transcellular potassium ion shift Insulin (regular) 10 U IV or weight basedc Typically administered with 25-50 g of IV glucose β2-adrenergic agonist Albuterol 5-20 mg, nebulized Watch for tachycardia Sodium bicarbonate (NaHCO3) 50 mEq/50 mL IV Controversial outside of setting of severe metabolic acidosis; bolus dose is very hypertonic with significant sodium load Removal from body Loop diuretic Furosemide 40-60 mg IV Supplement with isotonic saline solution if euvolemic or hypovolemic; highly effective if the patient is diuretic-responsive Cation exchange resins Sodium polystyrene sulfonate, 15 g orally/ rectally, 1-4×/d Use with caution with ileus/obstruction or in patients with dehydration; associated with risk for colonic necrosis; need to ensure that the resin transits out of the GI tract; other resins are under development but are not approved for use for acute hyperkalemia Abbreviations: AKI, acute kidney injury; ECG, electrocardiographic; GI, gastrointestinal; IV, intravenous. a ECG changes include peaked T waves, prolongation of PR interval, widening of QRS, second- or third-degree heart block, and sine wave pattern (from least severe to most life-threatening). b For AKI with hyperkalemia in the setting of digoxin toxicity, reversal of digoxin toxicity with digoxin antibody fragments is the treatment of choice. c Weight-based insulin dosing is 0.1 U/kg of body weight, up to 10 units. It is associated with reduced risk for hypoglycemia without affecting potassium-lowering effect. 144 AJKD Vol 72 | Iss 1 | July 2018 Core Curriculum
  • 10. Consequently, current guidelines recommend goal effluent flow rates of 20 to 25 mL/kg/h. With regard to IHD dosing, it is important to routinely check the urea reduction ratio or Kt/V to ensure that dialysis is adequate. In the VA/NIH ATN study, median duration of an IHD session was 4 hours, with a mean blood flow rate of 360 mL/min, highlighting that in these catabolic patients, substantial time is needed to ensure an adequate dialysis dose. Discontinuation of RRT The decision to discontinue RRT in patients with AKI is made based on 1 of 3 clinical scenarios: intrinsic kidney function has adequately improved to meet demands, the disorder that prompted renal support has improved, or continued RRT is no longer consistent with goals of care. There is no definitive prospective evidence to guide cli- nicians, but urine output appears to be predictive of successful RRT discontinuation. In one study of patients on CRRT, 24-hour urine output > 400 mL/d in patients not using diuretics or >2,300 mL/d in patients using diuretics had >80% chance of successful RRT discontinuation. Other studies have suggested that quantitation of timed urinary creatinine and urea excretion (either as total excretion per 24-hour period or calculation of creatinine and urea clearance) may be helpful. Prospective studies are needed to help guide clinicians on when to attempt RRT discontinuation. Additional Readings ► Gaudry S, Hajage D, Schortgen F, et al. Initiation strategies for renal-replacement therapy in the intensive care unit. N Engl J Med. 2016;375:122-133. ► Macedo E, Mehta RL. Continuous dialysis therapies: core cur- riculum 2016. Am J Kidney Dis. 2016;68:645-657. + ESSENTIAL READING Figure 1. Conceptual model for the diagnosis and treatment of acute kidney injury (AKI). Abbreviations: UO, urine output; RRT, renal replacement therapy; sCr, serum creatinine. AJKD Vol 72 | Iss 1 | July 2018 145 Core Curriculum
  • 11. ► Ponce D, Balbi A, Cullis B. Acute PD: evidence, guidelines, and controversies. Semin Nephrol. 2017;37:103-112. ► RENAL Replacement Therapy Study Investigators’ Bellomo R, Cass A, Cole L, et al. Intensity of continuous renal-replacement therapy in critically ill patients. N Engl J Med. 2009;361: 1627-1638. ► Uchino S, Bellomo R, Morimatsu H, et al. Discontinuation of continuous renal replacement therapy: a post hoc analysis of a prospective multicenter observational study. Crit Care Med. 2009;37:2576-2582. ► VA/NIH Acute Renal Failure Trial Network; Palevsky PM, Zhang JH, O’Connor TZ, et al. Intensity of renal support in critically ill patients with AKI. N Engl J Med. 2008;359:7-20. + ESSENTIAL READING ► Wald R, Adhikari NK, Smith OM, et al. Comparison of standard and accelerated initiation of renal replacement therapy in AKI. Kidney Int. 2015;88:897-904. ► Wheeler DT, Schafers SJ, Horwedel TA, Deal EN, Tobin GS. Weight-based insulin dosing for acute hyperkalemia results in less hypoglycemia. J Hosp Med. 2016;11:355-357. ► Zarbock A, Kellum JA, Schmidt C, et al. Effect of early vs delayed initiation of renal replacement therapy on mortality in critically ill patients with AKI: the ELAIN randomized clinical trial. JAMA. 2016;315:2190-2199. Contrast-Induced AKI Contrast-induced AKI (CI-AKI, also referred to as contrast- associated AKI) is a specific form of AKI that usually manifests as a transient small increase in Scr concentration within a few days of exposure to intravascular iodinated contrast. Despite its usually self-limited course, CI-AKI is associated with increased short- and long-term mortality, as well as progressive CKD. Recently, the degree to which radiocontrast affects the kidney has been debated because several studies (both meta-analyses and cohort studies) have suggested that in the aggregate population, the risk for AKI after contrast administration is perhaps overemphasized. Nonetheless, in clinical practice, for any given study requiring iodinated contrast, the potential risks and benefits should be weighed closely. Along the same lines, patient- and procedure-level factors contribute to the risk for CI-AKI and should be assessed. The primary risk factor for CI-AKI is CKD, and the incidence of CI- AKI increases incrementally as GFR decreases or pro- teinuria/albuminuria increases. Diabetes further in- creases the risk in those with CKD. Additional patient- specific risk factors include low effective circulating blood volume and nonsteroidal anti-inflammatory drug use. Procedure-related risk factors include higher contrast volume, intra-arterial procedures, multiple contrast exposures in a short interval, and hyperosmolar contrast agents. Management of CI-AKI aims primarily at prevention. Consideration should be given to alternative noncontrast studies if possible. Those who undergo iodinated contrast studies should have treatment with nonste- roidal anti-inflammatory drugs and other nephrotoxins discontinued, ideally at least 24 hours before the pro- cedure. Low- or iso-osmolar radiocontrast should be used, at the lowest possible volume required. Isotonic intravenous fluid administration reduces the risk for CI- AKI and should be used in those at elevated risk. Typical regimens consist of a 1-mL/kg/h infusion 12 hours before and 12 hours after contrast exposure, or 3 mL/kg/ h 1 hour before and 1.5 mL/kg/h for 4 to 6 hours postprocedure. With regard to fluid selection, although small studies suggested a benefit to the use of isotonic sodium bicarbonate solution, a large randomized clinical trial of isotonic bicarbonate versus normal saline solution (factorialized with N-acetylcysteine vs placebo) in high- risk patients undergoing angiography showed no benefit with bicarbonate or N-acetylcysteine with regard to a composite end point of death, RRT, and 50% reduction in GFR at 90 days. There have been a variety of other pharmacotherapies evaluated for CI-AKI prevention, none of which is clearly beneficial. Hemodialysis after admin- istration of contrast is ineffective for preventing CI-AKI and may cause harm. Additional Readings ► James MT, Samuel SM, Manning MA, et al. Contrast-induced AKI and risk of adverse clinical outcomes after coronary angiography: a systematic review and meta-analysis. Circ Cardiovasc Interv. 2013;6:37-43. ► Weisbord SD, Gallagher M, Jneid H, et al. Outcomes after angiography with sodium bicarbonate and acetylcysteine [published online ahead of print November 12, 2017]. N Engl J Med. doi: 10.1056/NEJMoa1710933. + ESSENTIAL READING ► Weisbord SD, Palevsky PM. Prevention strategies for contrast- induced nephropathy. Ann Intern Med. 2016;164:511. + ESSENTIAL READING ► Wilhelm-Leen E, Montez-Rath ME, Chertow G. Estimating the risk of radiocontrast-associated nephropathy. J Am Soc Nephrol. 2017;28:653-659. Long-term Consequences of AKI Case, continued: Two weeks later, the patient begins to recover kidney function. He is discharged from the hospital with an Scr concentration that is stable at 2.5 mg/dL. He asks you about the long-term impact of the AKI on his health. Question 4: What is the best way to respond to his stated concern? a) Because this was an acute event due to urosepsis, which is now fully treated, the AKI has no meaningful impact on the course of his underlying CKD b) His risk for future dialysis dependency has increased significantly after this episode of AKI c) There is no association between his recent AKI and risk for future cardiovascular disease d) He should expect further recovery of his kidney function and return to his baseline over the next few months For answer, see Appendix. 146 AJKD Vol 72 | Iss 1 | July 2018 Core Curriculum
  • 12. Although previously it was believed that most pa- tients who developed AKI fully recovered, it is now recognized that those who experience AKI have increased risk for subsequent AKI, progressive CKD, and future mortality. Even mild stages of AKI are associated with incident CKD. In a propensity-matched cohort study of hospitalized patients who experienced renal recovery based on Scr concentration, those with AKI had an increased rate of incident CKD (relative risk [RR], 2.14; 95% CI, 1.96-2.43) and mortality (RR, 1.48; 95% CI, 1.20-1.83). A pooled analysis of studies of long-term risk for CKD and dialysis dependence found a pooled hazard ratio of 8.8 for CKD and 3.1 for end-stage kidney dis- ease in patients with AKI compared with those without AKI. There was a graded increase in risk by severity of AKI. Given the retrospective nature of these associa- tions, it is controversial whether this is a causal rela- tionship or the development of AKI is simply a marker of those at higher risk for CKD. An ongoing matched cohort study sponsored by the National Institute of Diabetes and Digestive and Kidney Diseases (NIDDK) is focusing on individuals who survive 3 months after a hospitalization with or without AKI and is designed to try to address some of these remaining questions. Furthermore, identifying renal recovery based on Scr concentration may be difficult because hospitalized patients are at risk for muscle mass loss, creatinine production can by decreased by inflammation, and Scr can by diluted by iatrogenic volume overload. This was demonstrated by Prowle et al, who found that Scr concentrations were lower on discharge than on admission in ICU patients without AKI. Using a model taking into account this decrease in Scr concentration, significantly more patients with AKI would have had continued decreased kidney function compared with estimates calculated from unadjusted discharge Scr concentrations. Apart from CKD and death, there has been considerable interest in AKI as a risk factor for cardiovascular disease events. A recent meta-analysis showed that AKI was associated with a 58% increased risk for subsequent heart failure events and 40% increased risk for acute myocardial infarction. However, because most studies were con- ducted in patients with pre-existing cardiovascular disease, further research is needed to elucidate potential mecha- nisms by which AKI contributes to CVD. One potential mechanism is through hypertension. A recent study of more than 40,000 hospitalized adult patients without known hypertension showed that an episode of in- hospital AKI was strongly predictive of subsequent hy- pertension within 2 years (adjusted OR, 1.22; 95% CI, 1.12-1.33). It is currently recommended that all patients who experience AKI have their kidney function re-evaluated 3 months after AKI to identify those with new/worsening CKD, which should be managed accordingly. Even those who return to their baseline kidney function should be considered at elevated risk for the development of CKD. At this time it is unclear whether any intervention or increase in monitoring would reduce the risk for poor outcomes in these patients. Additional Readings ► Bucaloiu ID, Kirchner HL, Norfolk ER, Hartle JE, Perkins RM. Increased risk of death and de novo CKD following reversible AKI. Kidney Int. 2012;81:477-485. + ESSENTIAL READING ► Chawla LS, Eggers PW, Star RA, Kimmel PL. AKI and CKD as interconnected syndromes. N Engl J Med. 2014;371:58-66. + ESSENTIAL READING ► Coca SG, Singanamala S, Parikh CR. CKD after AKI: a system- atic review and meta-analysis. Kidney Int. 2012;81:442-448. ► Hsu CY, Hsu RK, Yang J, Ordonez JD, Zheng S, Go AS. Elevated BP after AKI. J Am Soc Nephrol. 2016;27:914-923. ► Odutayo A, Wong CX, Farkouh M, et al. AKI and long-term risk for cardiovascular events and mortality. J Am Soc Nephrol. 2017;28:377-387. ► Parr SK, Siew ED. Delayed consequences of AKI. Adv Chronic Kidney Dis. 2016;23:186-194. ► Prowle JR, Kolic I, Purdell-Lewis J, Taylor R, Pearse RM, Kirwan CJ. Serum creatinine changes associated with critical illness and detection of persistent renal dysfunction after AKI. Clin J Am Soc Nephrol. 2014;9:1015-1023. Article Information Authors’ Full Names and Academic Degrees: Peter K. Moore, MD, Raymond K. Hsu, MD, MAS, and Kathleen D. Liu, MD, PhD, MAS. Authors’ Affiliations: Division of Hospital Medicine, Department of Medicine, San Francisco Veterans Affairs Medical Center and University of California San Francisco (PKM); and Division of Nephrology, Department of Medicine (RKS, KDL), and Critical Care Medicine, Department of Anesthesia (KDL), University of California, San Francisco, CA. Address for Correspondence: Kathleen D. Liu, MD, PhD, MAS, Division of Nephrology, Box 0532, University of California, San Francisco, San Francisco, CA 94143-0532.. E-mail: kathleen.liu@ ucsf.edu Support: None. Financial Disclosure: Dr Hsu reports funding from the NIDDK and has been a consultant for Retrophin. Dr Liu has funding from the NIDDK and the National Heart, Lung and Blood Institute; has been a consultant for Achaogen, Durect, Quark, Potrero Medical and Theravance; has served on an Advisory Board for ZS Pharma; and holds stock in Amgen. None of these financial disclosures are relevant to the current article. Acknowledgements: The authors appreciate the assistance of Dr Asghar Rastegar with the case that is part of this Core Curriculum. Peer Review: Received August 3, 2017 in response to an invitation from the journal. Evaluated by 3 external peer reviewers, with direct editorial input from the Feature Editor, the Education Editor, and a Deputy Editor. Accepted in revised form November 19, 2017. AJKD Vol 72 | Iss 1 | July 2018 147 Core Curriculum
  • 13. APPENDIX Answer to Question 1: (d) Because his Scr concentration is increasing, none of the standard formulas should be used to estimate his kidney function. Answer to Question 2: He is fluid overloaded, as shown by his markedly positive fluid balance and the presence of edema. At this point, additional fluid should be adminis- tered with caution because it will likely only exacerbate fluid overload. MAP is 75 mm Hg and therefore the addition of a vasopressor is not justified. Use of furosemide may increase urine output and decrease fluid overload; however, it probably would not change the overall clinical outcome. Thus, (d) is the best answer. Answer to Question 3: (c) There is no evidence that CRRT has a special role in patients with sepsis-associated AKI. With regard to CRRT dose, 20 to 25 mL/kg/h has similar outcomes to higher doses of therapy. The major indication for RRT is the lack of renal recovery in association with significant fluid overload and progressive hypoxemia. Answer to Question 4: (b) AKI is a risk factor for CKD progression and end-stage kidney disease, as well as car- diovascular events (see text for full discussion). It is un- known whether he will have further improvement in kidney function over time, but it seems unlikely because his Scr concentration is now stable. 148 AJKD Vol 72 | Iss 1 | July 2018 Core Curriculum